NR509 Midterm Exam Test Bank_ Fall 2022 Pass your exams in only 1 attempt.

A patient presents for evaluation of a sharp, aching chest pain which increases with breathing. Which anatomic area would you localize the symptom to?

A) Musculoskeletal
B) Reproductive
C) Urinary
D) Endocrine
A) Musculoskeletal

Chest pain may be due to a musculoskeletal condition, such as costochondritis or
intercostal muscle cramp. This would be worsened by motion of the chest wall. Pleuritic chest pain is also a sharp chest pain which increases with a deep breath. This type of pain can
occur with inflammation of the pleura from pneumonia or other conditions and pulmonary
embolus.

A patient comes to the emergency room for evaluation of shortness of breath. To which anatomic region would you assign the symptom?

A) Reproductive
B) Urinary
C) Cardiac
D) Hematologic
C) Cardiac

Cardiac disorders such as congestive heart failure are the most likely on this list to result in shortness of breath. There are cases within the other categories which may also result
in shortness of breath, such as anemia in the hematologic category, pregnancy in the reproductive
category, or sepsis with UTI in the urinary category.

A patient presents for evaluation of a cough. Which of the following anatomic regions can be responsible for a cough?

A) Ophthalmologic
B) Auditory
C) Cardiac
D) Endocrine
C) Cardiac

The cardiac system can cause a cough if the patient has congestive heart failure. This results in fluid buildup in the lungs, which in turn can cause a cough that produces pink,
frothy sputum. A foreign body in the ear may also cause a cough by stimulating Arnold’s branch of the vagus nerve, but this is less likely to be seen clinically than heart failure.

A 22-year-old advertising copywriter presents for evaluation of joint pain. The pain is new,
located in the wrists and fingers bilaterally, with some subjective fever. The patient denies a
rash; she also denies recent travel or camping activities. She has a family history significant for
rheumatoid arthritis. Based on this information, which of the following pathologic processes
would be the most correct?

A) Infectious
B) Inflammatory
C) Hematologic
D) Traumatic
B) Inflammatory

The description is most consistent with an inflammatory process, although all the other etiologies should be considered. Lyme disease is an infection which commonly causes
arthritis, hemophilia is a hematologic condition which can cause bleeding in the joints, and trauma can obviously cause joint pain.

A 47-year-old contractor presents for evaluation of neck pain, which has been intermittent for several years. He normally takes over-the-counter medications to ease the pain, but this time they haven’t worked as well and he still has discomfort. He recently wallpapered the entire second floor in his house, which caused him great discomfort. The pain resolved with rest. He denies fever, chills, rash, upper respiratory symptoms, trauma, or injury to the neck. Based on this description, what is the most likely pathologic process?

A) Infectious
B) Neoplastic
C) Degenerative
D) Traumatic
C) Degenerative

The description is most consistent with degenerative arthritis in the neck. The
patient has had intermittent symptoms and the questions asked to elicit pertinent negative and
positive findings are negative for infectious, traumatic, or neoplastic disease.

A 15-year-old high school sophomore comes to the clinic for evaluation of a 3-week history
of sneezing; itchy, watery eyes; clear nasal discharge; ear pain; and nonproductive cough. Which
is the most likely pathologic process?

A) Infection
B) Inflammation
C) Allergic
D) Vascular
C) Allergic

This description is most consistent with allergic rhinitis.

A 19-year old-college student presents to the emergency room with fever, headache, and
neck pain/stiffness. She is concerned about the possibility of meningococcal meningitis.
Several of her dorm mates have been vaccinated, but she hasn’t been. Which of the following
physical examination descriptions is most consistent with meningitis?

A) Head is normocephalic and atraumatic, fundi with sharp discs, neck supple with full range
of motion
B) Head is normocephalic and atraumatic, fundi with sharp discs, neck with paraspinous
muscle spasm and limited range of motion to the right
C) Head is normocephalic and atraumatic, fundi with blurred disc margins, neck tender to
palpation, unable to perform range of motion
D) Head is normocephalic and atraumatic, fundi with blurred disc margins, neck supple with
full range of motion
C) Head is normocephalic and atraumatic, fundi with blurred disc margins, neck tender to
palpation, unable to perform range of motion

Blurred disc margins are consistent with papilledema, and neck tenderness and lack of range of motion are consistent with neck stiffness, which in this scenario is likely to be caused
by meningeal inflammation. Kernig’s and Brudzinski’s signs are also helpful in testing for meningeal irritation on exam.

A 37-year-old nurse comes for evaluation of colicky right upper quadrant abdominal pain. The pain is associated with nausea and vomiting and occurs 1 to 2 hours after eating greasy foods. Which one of the following physical examination descriptions would be most consistent with the diagnosis of cholecystitis?

A) Abdomen is soft, nontender, and nondistended, without hepatosplenomegaly or masses.
B) Abdomen is soft and tender to palpation in the right lower quadrant, without rebound or
guarding.
C) Abdomen is soft and tender to palpation in the right upper quadrant with inspiration, to the
point of stopping inspiration, and there is no rebound or guarding.
D) Abdomen is soft and tender to palpation in the mid-epigastric area, without rebound or
guarding.
C) Abdomen is soft and tender to palpation in the right upper quadrant with inspiration, to the
point of stopping inspiration, and there is no rebound or guarding.

In cholecystitis, the pain, which originates from the gallbladder, is located in the
right upper quadrant. Severity of pain with inspiration that is sufficient to stop further inhalation
is also known as Murphy’s sign, which, if present, is further indicative of inflammation of the
gallbladder.

A 55-year-old data entry operator comes to the clinic to establish care. She has the following
symptoms: headache, neck pain, sinus congestion, sore throat, ringing in ears, sharp brief chest
pains at rest, burning abdominal pain with spicy foods, constipation, urinary frequency that is worse with coughing and sneezing, and swelling in legs. This cluster of symptoms is explained
by:

A) One disease process
B) More than one disease process
B) More than one disease process

The patient appears to have several possible conditions: allergic rhinitis, arthritis,
conductive hearing loss, pleuritic chest pains, heartburn, stress urinary incontinence, and venous stasis, among other conditions. Although we always try, it is very difficult to assign all of these
symptoms to one cohesive diagnosis.

A 62-year-old teacher presents to the clinic for evaluation of the following symptoms: fever, headache, sinus congestion, sore throat, green nasal discharge, and cough. This cluster of symptoms is best explained by:

A) One disease process
B) More than one disease process
A) One disease process

This cluster of symptoms is most consistent with sinusitis. The chance that all of these symptoms are caused by multiple synchronous conditions in the same patient is much less than the possibility of having one problem which accounts for all of them.

Steve has just seen a 5-year-old girl who wheezes when exposed to cats. The patient’s family history is positive for asthma. You think the child most likely has asthma. What have you just accomplished?

A) You have tested your hypothesis.
B) You have developed a plan.
C) You have established a working diagnosis.
D) You have created a hypothesis.
D) You have created a hypothesis.

As you go through a history and examination, you will start to generate ideas to explain the patient’s symptoms. It is best to keep an open mind and make as many hypotheses
as you can, to avoid missing a possibility. A common mistake is to latch onto one idea too early.

Ms. Washington is a 67-year-old who had a heart attack last month. Now she complains of shortness of breath and not being able to sleep in a flat position (orthopnea). On examination you note increased jugular venous pressure, an S3 gallop, crackles low in the lung fields, and swollen ankles (edema). This is an example of a:

A) Pathophysiologic problem
B) Psychopathologic problem
A) Pathophysiologic problem

This is an example of a pathophysiologic problem because Ms. Washington’s
symptoms are consistent with a pathophysiologic process. The heart attack reduced the ability of her heart to handle her volume status and subsequently produced the many features of congestive heart failure.

On the way to see your next patient, you glance at the calendar and make a mental note to
buy a Mother’s Day card. Your patient is Ms. Hernandez, a 76-year-old widow who lost her
husband in May, two years ago. She comes in today with a headaches, abdominal pain, and
general malaise. This happened once before, about a year ago, according to your detailed office
notes. You have done a thorough evaluation but are unable to arrive at a consistent picture to
tie these symptoms together. This is an example of a:

A) Pathophysiologic problem
B) Psychopathologic problem
B) Psychopathologic problem

It is not uncommon for patients to experience psychopathologic symptoms around
the anniversary of a traumatic event. The time of year and the lack of an obvious connection
between Ms. Hernandez’s symptoms would make you consider this as a possibility. You will note that although this might have been an early consideration in your hypothesis generation, it is
key to convince yourself that there is not a physiologic explanation for these symptoms, by
performing a careful history and examination.

Mr. Larson is a 42-year-old widowed father of two children, ages 4 and 11. He works in a
sales office to support his family. Recently he has injured his back and you are thinking he
would benefit from physical therapy, three times a week, for an hour per session. What would
be your next step?

A) Write the physical therapy prescription.
B) Have your office staff explain directions to the physical therapy center.
C) Discuss the plan with Mr. Larson.
D) Tell Mr. Larson that he will be going to physical therapy three times a week.
C) Discuss the plan with Mr. Larson.

You should discuss your proposed plan with the patient before implementing it. In this case, you and Mr. Larson will need to weigh the benefit of physical therapy against the
ability to provide for his family. You may need to consider other ways of helping the patient,
perhaps through prescribed back exercises he can do at home. It is a common mistake to
implement a plan without coming to an agreement with the patient first.

You are seeing an elderly man with multiple complaints. He has chronic arthritis, pain from an old war injury, and headaches. Today he complains of these pains, as well as dull chest pain under his sternum. What would the order of priority be for your problem list?

A) Arthritis, war injury pain, headaches, chest pain
B) War injury pain, arthritis, headaches, chest pain
C) Headaches, arthritis, war injury pain, chest pain
D) Chest pain, headaches, arthritis, war injury pain
D) Chest pain, headaches, arthritis, war injury pain

The problem list should have the most active and serious problem first. This new complaint of chest pain is almost certainly a higher priority than his other, more chronic
problems.

You are excited about a positive test finding you have just noticed on physical examination
of your patient. You go on to do more examination, laboratory work, and diagnostic tests, only
to find that there is no sign of the disease you thought would correlate with the finding. This
same experience happens several times. What should you conclude?

A) Consider not doing this test routinely.
B) Use this test when you have a higher suspicion for a certain correlating condition.
C) Continue using the test, perhaps doing less laboratory work and diagnostics.
D) Omit this test from future examinations.
C) Continue using the test, perhaps doing less laboratory work and diagnostics.

This is an example of a specific test that lacks sensitivity. With this scenario,
when you finally find a positive, you might be very certain that a given condition is present.
We generally develop our examinations to fit our clinical experiences. Sensitive tests are
performed routinely on the screening examination, while specific tests are usually saved for the
detailed or “branched” examinations. Branched examinations are further maneuvers we can
perform to investigate positive findings on our screening examinations. Save this type of
maneuver to confirm your hypothesis.

You have recently returned from a medical missions trip to sub-Saharan Africa, where you
learned a great deal about malaria. You decide to use some of the same questions and
maneuvers in your “routine” when examining patients in the midwestern United States. You
are disappointed to find that despite getting some positive answers and findings, on further
workup, none of your patients has malaria except one, who recently emigrated from Ghana. How should you next approach these questions and maneuvers?

A) Continue asking these questions in a more selective way.
B) Stop asking these questions, because they are low yield.
C) Question the validity of the questions.
D) Ask these questions of all your patients.
A) Continue asking these questions in a more selective way.

The predictive value of a positive finding depends upon the prevalence of a given
disease in a population. The prevalence of malaria in the Midwest is almost zero, except in people immigrating from areas of high prevalence. You will waste time and resources applying
these questions and maneuvers to all patients. It would be wise to continue applying what you
learned to those who are from areas of high prevalence of a given disease. You will learn to tailor your examination to the population you
are serving.

For which of the following patients would a comprehensive health history be appropriate?

A) A new patient with the chief complaint of “I sprained my ankle”.
B) An established patient with the chief complaint of “I have an upper respiratory infection”.
C) A new patient with the chief complaint of “I am here to establish care”.
D) A new patient with the chief complaint of “I cut my hand”.
C) A new patient with the chief complaint of “I am here to establish care”.

This patient is here to establish care, and because she is new to you, a
comprehensive health history is appropriate.

The components of the health history include all of the following except which one?

A) Review of systems
B) Thorax and lungs
C) Present illness
D) Personal and social items
B) Thorax and lungs

The thorax and lungs are part of the physical examination, not part of the health
history.

Is the following information subjective or objective?
Mr. M. has shortness of breath that has persisted for the past 10 days; it is worse with activity
and relieved by rest.

A) Subjective
B) Objective
A) Subjective

This is information given by the patient about the circumstances of his chief
complaint. It does not represent an objective observation by the examiner.

Is the following information subjective or objective?
Mr. M. has a respiratory rate of 32 and a pulse rate of 120.

A) Subjective
B) Objective
B) Objective

This is a measurement obtained by the examiner, so it is considered objective data. The patient is unlikely to be able to give this information to the examiner.

The following information is recorded in the health history: “The patient has had abdominal
pain for 1 week. The pain lasts for 30 minutes at a time; it comes and goes. The severity is 7 to 9
on a scale of 1 to 10. It is accompanied by nausea and vomiting. It is located in the
mid-epigastric area.”
Which of these categories does it belong to?

A) Chief complaint
B) Present illness
C) Personal and social history
D) Review of systems
B) Present illness

This information describes the problem of abdominal pain, which is the present
illness. The interviewer has obtained the location, timing, severity, and associated manifestations
of the pain. The interviewer will still need to obtain information concerning the quality of the
pain, the setting in which it occurred, and the factors that aggravate and alleviate the pain.

The following information is recorded in the health history: “The patient completed 8th
grade. He currently lives with his wife and two children. He works on old cars on the weekend.
He works in a glass factory during the week.”
Which category does it belong to?

A) Chief complaint
B) Present illness
C) Personal and social history
D) Review of systems
C) Personal and social history.

Personal and social history information includes educational level, family of origin,
current household status, personal interests, employment, religious beliefs, military history, and
lifestyle (including diet and exercise habits; use of alcohol, tobacco, and/or drugs; and sexual
preferences and history).

The following information is recorded in the health history: “I feel really tired.”
Which category does it belong to?

A) Chief complaint
B) Present illness
C) Personal and social history
D) Review of systems
A) Chief complaint

The chief complaint is an attempt to quote the patient’s own words. It is brief, like a headline, and further details should be sought in the present illness section.

The following information is recorded in the health history: “Patient denies chest pain,
palpitations, orthopnea, and paroxysmal nocturnal dyspnea.”
Which category does it belong to?

A) Chief complaint
B) Present illness
C) Personal and social history
D) Review of systems
D) Review of systems

Review of systems documents the presence or absence of common symptoms related to each major body system.

The following information is best placed in which category?
“The patient has had three cesarean sections.”

A) Adult illnesses
B) Surgeries
C) Obstetrics/gynecology
D) Psychiatric
B) Surgeries

A cesarean section is a surgical procedure. Approximate dates or the age of the
patient at the time of the surgery should also be recorded.

The following information is best placed in which category?
“The patient had a stent placed in the left anterior descending artery (LAD) in 1999.”

A) Adult illnesses
B) Surgeries
C) Obstetrics/gynecology
D) Psychiatric
A) Adult illnesses

The adult illnesses category is reserved for chronic illnesses, significant
hospitalizations, significant injuries, and significant procedures. A stent is a major procedure but does not involve a surgery.

The following information is best placed in which category?
“The patient was treated for an asthma exacerbation in the hospital last year; the patient has
never been intubated.”

A) Adult illnesses
B) Surgeries
C) Obstetrics/gynecology
D) Psychiatric
A) Adult illnesses

This is information about a significant hospitalization and should be placed in the adult illnesses section. If the patient is being seen for an asthma exacerbation, you may consider placing this info in the present illness section, because it relates to the chief
complaint at that visit.

You are running late after your quarterly quality improvement meeting at the hospital and
have just gotten paged from the nurses’ station because a family member of one of your patients
wants to talk with you about that patient’s care. You have clinic this afternoon and are
double-booked for the first appointment time; three other patients also have arrived and are
sitting in the waiting room. Which of the following demeanors is a behavior consistent with
skilled interviewing when you walk into the examination room to speak with your first clinic
patient?

A) Irritability
B) Impatience
C) Boredom
D) Calm
D) Calm

The appearance of calmness and patience, even when time is limited, is the hallmark of a skilled interviewer.

Suzanne, a 25 year old, comes to your clinic to establish care. You are the student preparing
to go into the examination room to interview her. Which of the following is the most logical
sequence for the patient-provider interview?
A) Establish the agenda, negotiate a plan, establish rapport, and invite the patient’s story.
B) Invite the patient’s story, negotiate a plan, establish the agenda, and establish rapport.
C) Greet the patient, establish rapport, invite the patient’s story, establish the agenda, expand
and clarify the patient’s story, and negotiate a plan.
D) Negotiate a plan, establish an agenda, invite the patient’s story, and establish rapport.
C) Greet the patient, establish rapport, invite the patient’s story, establish the agenda, expand
and clarify the patient’s story, and negotiate a plan.

This is the most productive sequence for the interview. Greeting patients and
establishing rapport allows them to feel more comfortable before “inviting” them to relate their
story. After hearing the patient’s story, together you establish the agenda regarding the most
important items to expand upon. At the end, together you negotiate the plan of diagnosis and
treatment.

Alexandra is a 28-year-old editor who presents to the clinic with abdominal pain. The pain is
a dull ache, located in the right upper quadrant, that she rates as a 3 at the least and an 8 at the
worst. The pain started a few weeks ago, it lasts for 2 to 3 hours at a time, it comes and goes, and
it seems to be worse a couple of hours after eating. She has noticed that it starts after eating
greasy foods, so she has cut down on these as much as she can. Initially it occurred once a week,
but now it is occurring every other day. Nothing makes it better. From this description, which of the seven attributes of a symptom has been omitted?

A) Setting in which the symptom occurs
B) Associated manifestations
C) Quality
D) Timing
B) Associated manifestations

The interviewer has not recorded whether or not the pain has been accompanied by
nausea, vomiting, fever, chills, weight loss, and so on. Associated manifestations are additional
symptoms that may accompany the initial chief complaint and that help the examiner to start
refining his or her differential diagnosis.

Jason is a 41-year-old electrician who presents to the clinic for evaluation of shortness of
breath. The shortness of breath occurs with exertion and improves with rest. It has been going on
for several months and initially occurred only a couple of times a day with strenuous exertion;
however, it has started to occur with minimal exertion and is happening more than a dozen times per day. The shortness of breath lasts for less than 5 minutes at a time. He has no cough, chest
pressure, chest pain, swelling in his feet, palpitations, orthopnea, or paroxysmal nocturnal dyspnea.
Which of the following symptom attributes was not addressed in this description?

A) Severity
B) Setting in which the symptom occurs
C) Timing
D) Associated manifestations
A) Severity

The severity of the symptom was not recorded by the interviewer, so we have no understanding as to how bad the symptom is for this patient. This allows the comparison of pain intensity before and after an intervention.

You are interviewing an elderly woman in the ambulatory setting and trying to get more
information about her urinary symptoms. Which of the following techniques is not a component
of adaptive questioning?

A) Directed questioning: starting with the general and proceeding to the specific in a manner
that does not make the patient give a yes/no answer
B) Reassuring the patient that the urinary symptoms are benign and that she doesn’t need to
worry about it being a sign of cancer
C) Offering the patient multiple choices in order to clarify the character of the urinary
symptoms that she is experiencing
D) Asking her to tell you exactly what she means when she states that she has a urinary tract
infection
B) Reassuring the patient that the urinary symptoms are benign and that she doesn’t need to
worry about it being a sign of cancer

Reassurance is not part of clarifying the patient’s story; it is part of establishing
rapport and empathizing with the patient.

Mr. W. is a 51-year-old auto mechanic who comes to the emergency room wanting to be
checked out for the symptom of chest pain. As you listen to him describe his symptom in more
detail, you say “Go on,” and later, “Mm-hmmm.” This is an example of which of the following skilled interviewing techniques?

A) Echoing
B) Nonverbal communication
C) Facilitation
D) Empathic response
C) Facilitation

This is an example of facilitation. Facilitation can be posture, actions, or words that
encourage the patient to say more.

Mrs. R. is a 92-year-old retired teacher who comes to your clinic accompanied by her
daughter. You ask Mrs. R. why she came to your clinic today. She looks at her daughter and
doesn’t say anything in response to your question. This is an example of which type of
challenging patient?

A) Talkative patient
B) Angry patient
C) Silent patient
D) Hearing-impaired patient
C) Silent patient

This is one example of a silent patient. There are many possibilities for this patient’s silence: depression, dementia, the manner in which you asked the question, and so on.

Mrs. T. comes for her regular visit to the clinic. She is on your schedule because her regular
provider is on vacation and she wanted to be seen. You have heard about her many times from
your colleague and are aware that she is a very talkative person. Which of the following is a
helpful technique to improve the quality of the interview for both the provider and the patient?

A) Allow the patient to speak uninterrupted for the duration of the appointment.
B) Briefly summarize what you heard from the patient in the first 5 minutes and then try to have her focus on one aspect of what she told you.
C) Set the time limit at the beginning of the interview and stick with it, no matter what occurs
in the course of the interview.
D) Allow your impatience to show so that the patient picks up on your nonverbal cue that the
appointment needs to end.
B) Briefly summarize what you heard from the patient in the first 5 minutes and then try to have her focus on one aspect of what she told you.

You can also say, “I want to make sure I take good care of this problem because it is very important. We may need to talk about the others at the next appointment. Is that okay with you?” This is a technique that can help you to change the subject but, at the same time,
validate the patient’s concerns; it also can provide more structure to the interview.

Mrs. H. comes to your clinic, wanting antibiotics for a sinus infection. When you enter the
room, she appears to be very angry. She has a raised tone of voice and states that she has been
waiting for the past hour and has to get back to work. She states that she is unimpressed by the
reception staff, the nurse, and the clinic in general and wants to know why the office wouldn’t
call in an antibiotic for her. Which of the following techniques is not useful in helping to calm
this patient?

A) Avoiding admission that you had a part in provoking her anger because you were late
B) Accepting angry feelings from the patient and trying not to get angry in return
C) Staying calm
D) Keeping your posture relaxed
A) Avoiding admission that you had a part in provoking her anger because you were late

In this scenario, the provider was 1 hour late in seeing the patient. The provider
should acknowledge that he was late and apologize for this, no matter the reason for being late. It often helps to acknowledge that a patient’s anger with you is understandable and that you might be angry in a similar situation.

A 23-year-old graduate student comes to your clinic for evaluation of a urethral discharge. As the provider, you need to get a sexual history. Which one of the following questions is inappropriate for eliciting the information?

A) Are you sexually active?
B) When was the last time you had intimate physical contact with someone, and did that
contact include sexual intercourse?
C) Do you have sex with men, women, or both?
D) How many sexual partners have you had in the last 6 months?
A) Are you sexually active?

This is inappropriate because it is too vague. Given the complaint, you should probably assume that he is sexually active. A specific sexual history will help you to assess this patient’s risk for other sexually transmitted infections.

Mr. Q. is a 45-year-old salesman who comes to your office for evaluation of fatigue. He has come to the office many times in the past with a variety of injuries, and you suspect that he has a problem with alcohol. Which one of the following questions will be most helpful in diagnosing this problem?

A) You are an alcoholic, aren’t you?
B) When was your last drink?
C) Do you drink 2 to 3 beers every weekend?
D) Do you drink alcohol when you are supposed to be working?
B) When was your last drink?

This is a good opening question that is general and neutral in tone; depending on the
timing, you will be able to ask for more specific information related to the patient’s last drink.
The others will tend to stifle the conversation because they are closed-ended questions.

On a very busy day in the office, Mrs. Donelan, who is 81 years old, comes for her usual visit for her blood pressure. She is on a low-dose diuretic chronically and denies any side effects. Her blood pressure is 118/78 today, which is well-controlled. As you are writing her script, she mentions that it is hard not having her husband Bill around anymore. What would you
do next?

A) Hand her the script and make sure she has a 3-month follow-up appointment.
B) Make sure she understands the script.
C) Ask why Bill is not there.
D) Explain that you will have more time at the next visit to discuss this.
C) Ask why Bill is not there.

Sometimes, the patient’s greatest need is for support and empathy. It would be inappropriate to ignore this comment today. She may have relied heavily upon Bill for care and
may be in danger. She may be depressed and even suicidal, but you will not know unless you
discuss this with her. Most importantly, you should empathize with her by saying something
like “It must be very difficult not to have him at home” and allow a pause for her to answer. You may also ask “What did you rely on him to do for you?”

A patient is describing a very personal part of her history very quickly and in great detail.
How should you react to this?

A) Write down as much as you can, as quickly as possible.
B) Ask her to repeat key phrases or to pause at regular intervals, so you can get almost every
word.
C) Tell her that she can go over the notes later to make sure they are accurate.
D) Push away from the keyboard or put down your pen and listen.
D) Push away from the keyboard or put down your pen and listen.

This is a common event in clinical practice. It is much more important to listen actively with good eye contact at this time than to document the story verbatim. You want to
minimize interruption (e.g., answer B). It is usually not appropriate to ask a patient to go over
the written notes, but it would be a good idea to repeat the main ideas back to her. You should be
certain she has completed her story before doing this.

You arrive at the bedside of an elderly woman who has had a stroke, affecting her entire
right side. She cannot speak (aphasia). You are supposed to examine her. You notice that
the last examiner left her socks at the bottom of the bed, and although sensitive areas are covered by a sheet, the blanket is heaped by her feet at the bottom of the bed. What would you do next?
A) Carry out your examination, focusing on the neurologic portion, and then cover her
properly.
B) Carry out your examination and let the nurse assigned to her “put her back together.”
C) Put her socks back on and cover her completely before beginning the evaluation.
D) Apologize for the last examiner but let the next examiner dress and cover her.
C) Put her socks back on and cover her completely before beginning the evaluation.

It is crucial to make an effort to make a patient comfortable. In this scenario, the
patient can neither speak nor move well. Take a moment to imagine yourself in her situation.
As a matter of respect as well as comfort, you should cover the patient appropriately and
consider returning a little later to do your examination if you feel she is cold.

When you enter your patient’s examination room, his wife is waiting there with him.
Which of the following is most appropriate?

A) Ask if it’s okay to carry out the visit with both people in the room.
B) Carry on as you would ordinarily. The permission is implied because his wife is in the
room with him.
C) Ask his wife to leave the room for reasons of confidentiality.
D) First ask his wife what she thinks is going on.
A) Ask if it’s okay to carry out the visit with both people in the room.

Even in situations involving people very familiar with each other, it is important to respect individual privacy. There is no implicit consent merely because he has allowed his wife
to be in the room with him. On the other hand, it is inappropriate to assume that his wife should leave the room. Remember, the patient is the focus of the visit, so it would be appropriate to
allow him to control who is in the room with him and inappropriate to address his wife first.

A patient complains of knee pain on your arrival in the room. What should your first sentence be after greeting the patient?

A) How much pain are you having?
B) Have you injured this knee in the past?
C) When did this first occur?
D) Could you please describe what happened?
D) Could you please describe what happened?

When looking into a complaint, it is best to start with an invitation for the patient to tell you in his or her own words. More specific questions should be used later in the interview to fill in any gaps.

You have just asked a patient how he feels about his emphysema. He becomes silent, folds his arms across his chest and leans back in his chair, and then replies, “It is what it is.”
How should you respond?

A) “You seem bothered by this question.”
B) “Next, I would like to talk with you about your smoking habit.”
C) “Okay, let’s move on to your other problems.”
D) “You have adopted a practical attitude toward your problem.”
A) “You seem bothered by this question.”

You have astutely noted that the patient’s body language changed at the time you
asked this question, and despite the patient’s response, you suspect there is more beneath the
surface. By sharing your observation and leaving a pause, he
may begin to talk about some issues which are very important to him.

A patient tells you about her experience with prolonged therapy for her breast cancer.
You comment, “That must have been a very trying time for you.” What is this an example of?

A) Reassurance
B) Empathy
C) Summarization
D) Validation
D) Validation

This is an example of validation to legitimize her emotional experience.

You are performing a young woman’s first pelvic examination. You make sure to tell her
verbally what is coming next and what to expect. Then you carry out each maneuver of the
examination. You let her know at the outset that if she needs a break or wants to stop, this is
possible. You ask several times during the examination, “How are you doing, Brittney?”
What are you accomplishing with these techniques?

A) Increasing the patient’s sense of control
B) Increasing the patient’s trust in you as a caregiver
C) Decreasing her sense of vulnerability
D) All of the above
D) All of the above

These techniques minimize the effects of transitions during an examination and
empower the patient. Especially during a sensitive examination, it is important to give the patient as much control as possible.

When using an interpreter to facilitate an interview, where should the interpreter be
positioned?

A) Behind you, the examiner, so that the lips of the patient and the patient’s nonverbal cues can
be seen
B) Next to the patient, so the examiner can maintain eye contact and observe the nonverbal
cues of the patient
C) Between you and the patient so all parties can make the necessary observations
D) In a corner of the room so as to provide minimal distraction to the interview
B) Next to the patient, so the examiner can maintain eye contact and observe the nonverbal
cues of the patient

Interpreters are invaluable in encounters where the examiner and patient do not
speak the same language, including encounters with the deaf. It should be noted that deaf
people from different regions of the world use different sign languages. The priority is for you
to have a good view of the patient.

A 15-year-old high school sophomore and her mother come to your clinic because the
mother is concerned about her daughter’s weight. You measure her daughter’s height and weight
and obtain a BMI of 19.5 kg/m2. Based on this information, which of the following is appropriate?

A) Refer the patient to a nutritionist and a psychologist because the patient is anorexic.
B) Reassure the mother that this is a normal body weight.
C) Give the patient information about exercise because the patient is obese.
D) Give the patient information concerning reduction of fat and cholesterol in her diet because
she is obese.
B) Reassure the mother that this is a normal body weight.

The patient has a normal BMI; the range for a normal BMI is 18.5 to 24.9 kg/m2. You may be able to give the patient and her mother the lower limit of normal in pounds for her
daughter’s height, or instruct her in how to use a BMI table.

A 25-year-old radio announcer comes to the clinic for an annual examination. His BMI is
26.0 kg/m2. He is concerned about his weight. Based on this information, what is appropriate
counsel for the patient during the visit?

A) Refer the patient to a nutritionist because he is anorexic.
B) Reassure the patient that he has a normal body weight.
C) Give the patient information about reduction of fat, cholesterol, and calories because he is overweight.
D) Give the patient information about reduction of fat and cholesterol because he is obese.
C) Give the patient information about reduction of fat, cholesterol, and calories because he is overweight.

The patient has a BMI in the overweight range, which is 25.0 to 29.9 kg/m2. It is
prudent to give him information about reducing calories, fat, and cholesterol in his diet to help prevent further weight gain.

A 30-year-old sales clerk comes to your office wanting to lose weight; her BMI is 30.0
kg/m2. What is the most appropriate amount for a weekly weight reduction goal?

A) .5 to 1 pound per week
B) 1 to 2.5 pounds per week
C) 2.5 to 3.5 pounds per week
D) 3.5 to 4.5 pounds per week
A) .5 to 1 pound per week

Based on the NIH Obesity Guidelines, this is the weekly weight loss goal to strive for to maintain long-term control of weight. More rapid weight loss than this does not result in
a better outcome at one year.

A 67-year-old retired janitor comes to the clinic with his wife. She brought him in because
she is concerned about his weight loss. He has a history of smoking 3 packs of cigarettes a day
for 30 years, for a total of 90 pack-years. He has noticed a daily cough for the past several
years, which he states is productive of sputum. He came into the clinic approximately 1 year ago, and at that time his weight was 140 pounds. Today, his weight is 110 pounds.
Which one of the following questions would be the most important to ask if you suspect that he has lung cancer?

A) Have you tried to force yourself to vomit after eating a meal?
B) Do you have heartburn/indigestion and diarrhea?
C) Do you have enough food to eat?
D) Have you tried to lose weight?
D) Have you tried to lose weight?

This is important: If the patient hasn’t tried to lose weight, then this weight loss is inadvertent and poses concern for a neoplastic process, especially given his smoking history.

Common or concerning symptoms to inquire about in the General Survey and vital signs
include all of the following except:

A) Changes in weight
B) Fatigue and weakness
C) Cough
D) Fever and chills
C) Cough

This symptom is more appropriate to the respiratory review of systems.

You are beginning the examination of a patient. All of the following areas are important to
observe as part of the General Survey except:

A) Level of consciousness
B) Signs of distress
C) Dress, grooming, and personal hygiene
D) Blood pressure
D) Blood pressure

Blood pressure is a vital sign, not part of the General Survey.

A 55-year-old bookkeeper comes to your office for a routine visit. You note that on a previous visit for treatment of contact dermatitis, her blood pressure was elevated. She does not have prior elevated readings and her family history is negative for hypertension. You measure her blood pressure in your office today. Which of the following factors can result in a false high reading?

A) Blood pressure cuff is tightly fitted.
B) Patient is seated quietly for 10 minutes prior to measurement.
C) Blood pressure is measured on a bare arm.
D) Patient’s arm is resting, supported by your arm at her mid-chest level as you stand to
measure the blood pressure.
A) Blood pressure cuff is tightly fitted.

A blood pressure cuff that is too tightly fitted can result in a false high reading.
The other answers are important to observe to obtain an accurate blood pressure reading.
JNC-7 also mentions the importance of having the back supported when obtaining blood
pressure in the sitting position.

A 49-year-old truck driver comes to the emergency room for shortness of breath and
swelling in his ankles. He is diagnosed with congestive heart failure and admitted to the hospital. You are the student assigned to do the patient’s complete history and physical examination.
When you palpate the pulse, what do you expect to feel?

A) Large amplitude, forceful
B) Small amplitude, weak
C) Normal
D) Bigeminal
B) Small amplitude, weak

Congestive heart failure is characterized by decreased stroke volume or increased
peripheral vascular resistance, which would result in a small-amplitude, weak pulse. Subtle
differences in amplitude are usually best detected in large arteries close to the heart, like the
carotid pulse. You may not be able to notice these in other locations.

An 18-year-old college freshman presents to the clinic for evaluation of gastroenteritis. You
measure the patient’s temperature and it is 104 degrees Fahrenheit. What type of pulse would you
expect to feel during his initial examination?

A) Large amplitude, forceful
B) Small amplitude, weak
C) Normal
D) Bigeminal
A) Large amplitude, forceful

Fever results in an increased stroke volume, which results in a large-amplitude,
forceful pulse. Later in the course of the illness, if dehydration and shock result, you may expect small amplitude and weak pulses.

A 25-year-old type 1 diabetic clerk presents to the emergency room with shortness of breath and states that his blood sugar was 605 at home. You diagnose the patient with diabetic ketoacidosis. What is the expected pattern of breathing?

A) Normal
B) Rapid and shallow
C) Rapid and deep
D) Slow
C) Rapid and deep

The body is trying to
rid itself of carbon dioxide to compensate for the acidosis. This is known as Kussmaul’s
breathing and is seen in other causes of acidosis as well.

Mrs. Lenzo weighs herself every day with a very accurate balance-type scale. She has noticed that over the past 2 days she has gained 4 pounds. How would you best explain this?

A) Attribute this to some overeating at the holidays.
B) Attribute this to wearing different clothing.
C) Attribute this to body fluid.
D) Attribute this to instrument inaccuracy.
C) Attribute this to body fluid.

This amount of weight over a short period should make one think of body fluid changes. You may consider a kidney problem or heart failure in your differential. The other
reasons should be considered as well, but this amount of weight gain over a short period usually
indicates causes other than excessive caloric intake.

Mr. Curtiss has a history of obesity, diabetes, osteoarthritis of the knees, HTN, and obstructive sleep apnea. His BMI is 43 and he has been discouraged by his difficulty in losing weight. He is also discouraged that his goal weight is 158 pounds away. What would you
tell him?

A) “When you get down to your goal weight, you will feel so much better.”
B) “Some people seem to be able to lose weight and others just can’t, no matter how hard they
try.”
C) “We are coming up with new medicines and methods to treat your conditions every day.”
D) “Even a weight loss of 10% can make a noticeable improvement in the problems you
mention.”
D) “Even a weight loss of 10% can make a noticeable improvement in the problems you
mention.”

Many patients trying to change a habit are overwhelmed by how far they are from
their goal. Many
patients find it empowering to know that they can achieve a small goal, such as a loss of 1 pound
per week. Research has shown that significant benefits often come with even a 10% weight loss.

Jenny is one of your favorite patients who usually shares a joke with you and is nattily dressed. Today she is dressed in old jeans, lacks makeup, and avoids eye contact. To what do you attribute these changes?

A) She is lacking sleep.
B) She is fatigued from work.
C) She is running into financial difficulty.
D) She is depressed.
D) She is depressed.

It is important to use all of your skills and memory of an individual patient to guide your thought process. She is not described as sleepy. Work fatigue would most likely not
cause avoidance of eye contact. Financial difficulties would not necessarily deplete a nice wardrobe. It is most likely that she is depressed or in another type of difficulty.

You are seeing an older patient who has not had medical care for many years. Her vital signs taken by your office staff are: T 37.2, HR 78, BP 118/92, and RR 14, and she denies pain. You notice that she has some hypertensive changes in her retinas and you find mild proteinuria on a urine test in your office. You expected the BP to be higher. She is not on any
medications. What do you think is causing this BP reading, which doesn’t correlate with the
other findings?
A) It is caused by an “auscultatory gap.”
B) It is caused by a cuff size error.
C) It is caused by the patient’s emotional state.
D) It is caused by resolution of the process which caused her retinopathy and kidney problems.
A) It is caused by an “auscultatory gap.”

The blood pressure is unusual in this case in that the systolic pressure is normal
while the diastolic pressure is elevated. Especially with the retinal and urinary findings, you should consider that the BP may be much higher and that an auscultatory gap was missed. This can be avoided by checking for obliteration of the radial pulse while the cuff is inflated. Although a large cuff can cause a slightly lower BP on a patient with a small arm, this does not account for the elevated DBP.

Despite having high BP readings in the office, Mr. Kelly tells you that his readings at home are much lower. He checks them twice a day at the same time of day and has kept a log. How do you respond?

A) You diagnose “white coat hypertension.”
B) You assume he is quite nervous when he comes to your office.
C) You question the accuracy of his measurements.
D) You question the accuracy of your measurements.
C) You question the accuracy of his measurements.

It is not uncommon to see differences in a patient’s home measurements and your own in the office. Presuming that this is “white coat hypertension” can be dangerous because
this condition is not usually treated. This allows for the effects of a missed diagnosis of hypertension to go unchecked. It is also very difficult to judge if a patient is outwardly nervous. You should always consider that your measurements are not accurate as well, but the fact that you and your staff are well-trained and perform this procedure on hundreds of patients a week makes this less likely. Ideally, you would ask the patient to bring in his BP equipment and take a simultaneous reading with you to make sure that he is getting an accurate reading.

You are observing a patient with heart failure and notice that there are pauses in his breathing. On closer examination, you notice that after the pauses the patient takes
progressively deeper breaths and then progressively shallower breaths, which are followed by
another apneic spell. The patient is not in any distress. You make the diagnosis of:

A) Ataxic (Biot’s) breathing
B) Cheyne-Stokes respiration
C) Kussmaul’s respiration
D) COPD with prolonged expiration
B) Cheyne-Stokes respiration

Cheyne-Stokes respiration can be seen in patients with heart failure and is usually not a sign of an immediate problem.

Mr. Garcia comes to your office for a rash on his chest associated with a burning pain. Even a light touch causes this burning sensation to worsen. On examination, you note a rash with small blisters (vesicles) on a background of reddened skin. The rash overlies an entire rib
on his right side. What type of pain is this?

A) Idiopathic pain
B) Neuropathic pain
C) Nociceptive or somatic pain
D) Psychogenic pain
B) Neuropathic pain

This vignette is consistent with a diagnosis of herpes zoster, or shingles.
The characteristic burning quality without a history of an actual burn makes one think of
neuropathic pain. It will most likely remain for months after the rash has resolved. There is no
evidence of physical injury and this is a peculiar distribution, making nociceptive pain less likely. There is no evidence of a psychogenic etiology for this, and the presence of a rash
makes this possibility less likely as well.

A 50-year-old body builder is upset by a letter of denial from his life insurance company. He is very lean but has gained 2 pounds over the past 6 months. You personally performed his health assessment and found no problems whatsoever. He says he is classified as “high risk”
because of obesity. What should you do next?

A) Explain that even small amounts of weight gain can classify you as obese.
B) Place him on a high-protein, low-fat diet.
C) Advise him to increase his aerobic exercise for calorie burning.
D) Measure his waist.
D) Measure his waist.

The patient most likely had a high BMI because of increased muscle mass. In this
situation, it is important to measure his waist. It is most likely under 40 inches, which makes
obesity unlikely (even to an insurance company). It is important that you personally contact the company and explain your reasoning.

Ms. Wright comes to your office, complaining of palpitations. While checking her pulse you notice an irregular rhythm. When you listen to her heart, every fourth beat sounds different. It sounds like a triplet rather than the usual “lub dup.” How would you document
your examination?

A) Regular rate and rhythm
B) Irregularly irregular rhythm
C) Regularly irregular rhythm
D) Bradycardia
C) Regularly irregular rhythm

Because this unusual beat occurs every fourth set of heart sounds, it is regularly
irregular. This is most consistent with premature ventricular contractions (or PVCs). This is generally a common and benign rhythm.

A 19-year-old college student, Todd, is brought to your clinic by his mother. She is concerned that there is something seriously wrong with him. She states for the past 6 months his behavior has become peculiar and he has flunked out of college. Todd denies any recent illness or injuries. His past medical history is remarkable only for a broken foot. His parents are both
healthy. He has a paternal uncle who had similar symptoms in college. The patient admits to
smoking cigarettes and drinking alcohol. He also admits to marijuana use but none in the last
week. He denies using any other substances. He denies any feelings of depression or anxiety. While speaking with Todd and his mother you do a complete physical examination, which is essentially normal. When you question him on how he is feeling, he says that he is very worried that Microsoft has stolen his software for creating a better browser. He tells you he has seen a black van in his neighborhood at night and he is sure that it is full of computer tech workers stealing his work through special gamma waves. You ask him why he believes they are trying to
steal his programs. He replies that the technicians have been telepathing their intents directly into his head. He says he hears these conversations at night so he knows this is happening. Todd’s
mother then tells you, “See, I told you . . . he’s crazy. What do I do about it?”
While arranging for a psychiatry consult, what psychotic disorder do you think Todd has?

A) Schizoaffective disorder
B) Psychotic disorder due to a medical illness
C) Substance-induced psychotic disorder
D) Schizophrenia
D) Schizophrenia

Schizophrenia generally occurs in the late teens to early 20s. It often is seen in other
family members, as in this case. Symptoms must be present for at least 6 months and must have at least two features of (1) delusions (e.g., Microsoft is after his programs), (2) hallucinations
(e.g., technicians sending telepathic signals), (3) disorganized speech, (4) disorganized behavior, and (5) negative symptoms such as a flat affect.

A 24-year-old secretary comes to your clinic, complaining of difficulty sleeping, severe
nightmares, and irritability. She states it all began 6 months ago when she went to a fast food
restaurant at midnight. While she was waiting in her car a man entered through the passenger
door and put a gun to her head. He had her drive to a remote area, where he took her money and
threatened to kill her. When the gun jammed he panicked and ran off. Ever since this occurred
the patient has been having these symptoms. She states she jumps at every noise and refuses to
drive at night. She states her anxiety has had such a marked influence on her job performance she is afraid she will be fired. She denies any recent illnesses or injuries. Her past medical history is
unremarkable. On examination you find a nervous woman appearing her stated age. Her physical
examination is unremarkable. You recommend medication and counseling. What anxiety disorder to you think this young woman has?

A) Specific phobia
B) Acute stress disorder
C) Post-traumatic stress disorder
D) Generalized anxiety disorder
C) Post-traumatic stress disorder

Post-traumatic stress disorder is the fearful response (nightmares, avoidance of
areas, irritability) to an event that occurred at least 1 month prior to presentation. The patient’s fears and reactions cause marked distress and impair social and occupational functions.

A 75-year-old homemaker brings her 76-year-old husband to your clinic. She states that 4
months ago he had a stroke and ever since she has been frustrated with his problems with
communication. They were at a restaurant after church one Sunday when he suddenly became
quiet. When she realized something was wrong he was taken to the hospital by EMS. He spent 2
weeks in the hospital with right-sided weakness and difficulty speaking. After hospitalization he
was in a rehab center, where he regained the ability to walk and most of the use of his right hand.
He also began to speak more, but she says that much of the time “he doesn’t make any sense.”
She gives an example that when she reminded him the car needed to be serviced he told her “I
will change the Kool-Aid out of the sink myself with the ludrip.” She says that these sayings are
becoming frustrating. She wants you to tell her what is wrong and what you can do about it.
While you write up a consult to neurology, you describe the syndrome to her.
What type of aphasia does he have?

A) Wernicke’s aphasia
B) Broca’s aphasia
C) Dysarthria
A) Wernicke’s aphasia

With Wernicke’s aphasia the patient can speak effortlessly and fluently, but his words often make no sense. Words can be malformed or completely invented. Wernicke’s area is
found on the temporal lobes.

Susanne is a 27 year old who has had headaches, muscle aches, and fatigue for the last 2 months. You have completed a thorough history, examination, and laboratory workup but have
not found a cause. What would your next action be?

A) A referral to a neurologist
B) A referral to a rheumatologist
C) To tell the patient you can’t find anything
D) To screen for depression
D) To screen for depression

Although you may consider referrals to help with the diagnosis and treatment for
this patient, screening is a time-efficient way to recognize depression.

You ask a patient to draw a clock. He fills in all the numbers on the right half of the
circle. What do you suspect?

A) Hemianopsia
B) Fatigue
C) Oppositional defiant disorder
D) Depression
A) Hemianopsia

You should suspect a visual problem because there is no writing on one half of the circle. This is consistent with a hemianopsia, sometimes seen in stroke.

A 35-year-old archaeologist comes to your office (located in Phoenix, Arizona) for a regular
skin check-up. She has just returned from her annual dig site in Greece. She has fair skin and
reddish-blonde hair. She has a family history of melanoma. She has many freckles scattered
across her skin. From this description, which of the following is not a risk factor for melanoma in
this patient?

A) Age
B) Hair color
C) Actinic lentigines
D) Heavy sun exposure
A) Age

The risk for melanoma is increased in people over the age of 50; our patient is 35
years old. The other answers represent known risk factors for melanoma. Especially with a family history of melanoma, she should be instructed to keep her skin covered when in the sun and use strong sunscreen on exposed areas.

You are speaking to an 8th grade class about health prevention and are preparing to discuss the ABCDEs of melanoma. Which of the following descriptions correctly defines the ABCDEs?

A) A = actinic; B = basal cell; C = color changes, especially blue; D = diameter >6 mm; E = evolution
B) A = asymmetry; B = irregular borders; C = color changes, especially blue; D = diameter >6
mm; E = evolution
C) A = actinic; B = irregular borders; C = keratoses; D = dystrophic nails; E = evolution
D) A = asymmetry; B = regular borders; C = color changes, especially orange; D = diameter

6 mm; E = evolution
B) A = asymmetry; B = irregular borders; C = color changes, especially blue; D = diameter >6
mm; E = evolution

You are beginning the examination of the skin on a 25-year-old teacher. You have previously elicited that she came to the office for evaluation of fatigue, weight gain, and hair loss. You strongly suspect that she has hypothyroidism. What is the expected moisture and
texture of the skin of a patient with hypothyroidism?

A) Moist and smooth
B) Moist and rough
C) Dry and smooth
D) Dry and rough
D) Dry and rough

A patient with hypothyroidism is expected to have skin that is dry as well as rough.
This is a good example of how the skin can give clues to systemic diseases.

A 28-year-old patient comes to the office for evaluation of a rash. At first there was only one
large patch, but then more lesions erupted suddenly on the back and torso; the lesions itch. On
physical examination, you note that the pattern of eruption is like a Christmas tree and that there
are a variety of erythematous papules and macules on the cleavage lines of the back. Based on this description, what is the most likely diagnosis?

A) Pityriasis rosea
B) Tinea versicolor
C) Psoriasis
D) Atopic eczema
A) Pityriasis rosea

This is a classic description of pityriasis rosea. The description of a large single or “herald” patch preceding the eruption is a good way to distinguish this rash from other
conditions.

A 19-year-old construction worker presents for evaluation of a rash. He notes that it started
on his back with a multitude of spots and is also on his arms, chest, and neck. It itches a lot. He
does sweat more than before because being outdoors is part of his job. On physical examination,
you note dark tan patches with a reddish cast that has sharp borders and fine scales, scattered
more prominently around the upper back, chest, neck, and upper arms as well as under the arms. Based on this description, what is your most likely diagnosis?
A) Pityriasis rosea
B) Tinea versicolor
C) Psoriasis
D) Atopic eczema
B) Tinea versicolor

This is a typical description of tinea versicolor. The information that the patient is sweating more also helps support this diagnosis, because tinea is a fungal infection and is
promoted by moisture.

A 68-year-old retired farmer comes to your office for evaluation of a skin lesion. On the
right temporal area of the forehead, you see a flattened papule the same color as his skin, covered by a dry scale that is round and feels hard. He has several more of these scattered on the forehead, arms, and legs. Based on this description, what is your most likely diagnosis?

A) Actinic keratosis
B) Seborrheic keratosis
C) Basal cell carcinoma
D) Squamous cell carcinoma
This is a typical description of actinic keratosis. Actinic keratosis may be easier to feel than to see. If left untreated, approximately 1% of cases can develop into squamous cell
carcinoma.

A 58-year-old gardener comes to your office for evaluation of a new lesion on her upper chest. The lesion appears to be “stuck on” and is oval, brown, and slightly elevated with a flat surface. It has a rough, wartlike texture on palpation. Based on this description, what is your
most likely diagnosis?

A) Actinic keratosis
B) Seborrheic keratosis
C) Basal cell carcinoma
D) Squamous cell carcinoma
B) Seborrheic keratosis

This is a typical description for seborrheic keratosis. The “stuck on” appearance and
the rough, wartlike texture are key features for the diagnosis. They often produce a greasy scale when scratched with a fingernail, which further helps to distinguish them from other lesions.
Frequently, these benign lesions actually meet several of the ABCDEs of melanoma, so it is
important to distinguish these lesions to prevent unnecessary biopsy.

A 72-year-old teacher comes to a skilled nursing facility for rehabilitation after being in the
hospital for 6 weeks. She was treated for sepsis and respiratory failure and had to be on the
ventilator for 3 weeks. You are completing your initial assessment and are evaluating her skin
condition. On her sacrum there is full-thickness skin loss that is 5 cm in diameter, with damage
to the subcutaneous tissue. The underlying muscle is not affected. You diagnose this as a
pressure ulcer. What is the stage of this ulcer?
A) Stage 1
B) Stage 2
C) Stage 3
D) Stage 4
C) Stage 3

A stage 3 ulcer is a full-thickness skin loss with damage to or necrosis of
subcutaneous tissue that may extend to, but not through, the underlying muscle.

An 8-year-old girl comes with her mother for evaluation of hair loss. She denies pulling or twisting her hair, and her mother has not noted this behavior at all. She does not put her hair in braids. On physical examination, you note a clearly demarcated, round patch of hair loss without visible scaling or inflammation. There are no hair shafts visible. Based on this description, what is your most likely diagnosis?
A) Alopecia areata
B) Trichotillomania
C) Tinea capitis
D) Traction alopecia
A) Alopecia areata

This is a typical description for alopecia areata. There are no risk factors for
trichotillomania or for traction alopecia. The physical examination is not consistent with tinea capitis because the skin is intact.

A mother brings her 11 month old to you because her mother-in-law and others have told her that her baby is jaundiced. She is eating and growing well and performing the developmental milestones she should for her age. On examination you indeed notice a yellow tone to her skin from head to toe. Her sclerae are white. To which area should your next
questions be related?

A) Diet
B) Family history of liver diseases
C) Family history of blood diseases
D) Ethnicity of the child
A) Diet

The lack of jaundice in the sclerae is an important clue. Typically, this is the first place where one sees jaundice. This examination should also be carried out in natural light (sunlight) as opposed to fluorescent lighting, which can alter perceived colors. Many infants this age have a large proportion of carrots, tomatoes, and yellow squash, which are rich in
carotene. Liver and blood diseases can cause jaundice, but this should involve the sclerae.

A new mother is concerned that her child occasionally “turns blue.” On further questioning, she mentions that this is at her hands and feet. She does not remember the child’s lips turning blue. She is otherwise eating and growing well. What would you do now?

A) Reassure her that this is normal
B) Obtain an echocardiogram to check for structural heart disease and consult cardiology
C) Admit the child to the hospital for further observation
D) Question the validity of her story
A) Reassure her that this is normal.

This is an example of peripheral cyanosis. This is a very common and benign
condition which typically occurs when the child is slightly cold and his peripheral circulation is adjusting to keep his core warm. Without other problems, there is no need for further workup. If the lips or other central locations are involved, you must consider other etiologies.

You are examining an unconscious patient from another region and notice Beau’s lines, a transverse groove across all of her nails, about 1 cm from the proximal nail fold. What would you do next?

A) Conclude this is caused by a cultural practice.
B) Conclude this finding is most likely secondary to trauma.
C) Look for information from family and records regarding any problems which occurred 3
months ago.
D) Ask about dietary intake.
C) Look for information from family and records regarding any problems which occurred 3 months ago

These lines can provide valuable information about previous significant illnesses, some of which are forgotten or are not able to be reported by the patient. Because the
fingernails grow at about 0.1 mm per day, you would ask about an illness 100 days ago. Thisbpatient may have been hospitalized for endocarditis or may have had another significant illness
which should be sought. Trauma to all 10 nails in the same location is unlikely.

Dakota is a 14-year-old boy who just noticed a rash at his ankles. There is no history of
exposure to ill people or other agents in the environment. He has a slight fever in the office. The rash consists of small, bright red marks. When they are pressed, the red color remains.
What should you do?

A) Prescribe a steroid cream to decrease inflammation.
B) Consider admitting the patient to the hospital.
C) Reassure the parents and the patient that this should resolve within a week.
D) Tell him not to scratch them, and follow up in 3 days.
B) Consider admitting the patient to the hospital.

Although this may not be an impressive rash, the fact that they do not “blanch” with pressure is very concerning. This generally means that there is pinpoint bleeding under the
skin, and while this can be benign, it can be associated with life-threatening illnesses like
meningococcemia and low platelet counts (thrombocytopenia) associated with serious blood
disorders like leukemia.

Mrs. Hill is a 28-year-old African-American with a history of SLE (systemic lupus erythematosus). She has noticed a raised, dark red rash on her legs. When you press on the rash, it doesn’t blanch. What would you tell her regarding her rash?

A) It is likely to be related to her lupus.
B) It is likely to be related to an exposure to a chemical.
C) It is likely to be related to an allergic reaction.
D) It should not cause any problems.
A) It is likely to be related to her lupus.

A “palpable purpura” is usually associated with a vasculitis. This is an inflammatory condition of the blood vessels often associated with systemic rheumatic disease. It can cut off circulation to any portion of the body and can mimic many other diseases in this
manner. While allergic and chemical exposures may be a possible cause of the rash, this patient’s SLE should make you consider vasculitis.

Jacob, a 33-year-old construction worker, complains of a “lump on his back” over his scapula. It has been there for about a year and is getting larger. He says his wife has been able to squeeze out a cheesy-textured substance on occasion. He worries this may be cancer. When gently pinched from the side, a prominent dimple forms in the middle of the mass. What is
most likely?

A) An enlarged lymph node
B) A sebaceous cyst
C) An actinic keratosis
D) A malignant lesion
B) A sebaceous cyst

This is a classic description of an epidermal inclusion cyst resulting from a blocked sebaceous gland. The fact that any lesion is enlarging is worrisome, but the other descriptors
are so distinctive that cancer is highly unlikely. This would be an unusual location for a lymph
node, and these do not usually drain to the skin.

A young man comes to you with an extremely pruritic rash over his knees and elbows which has come and gone for several years. It seems to be worse in the winter and improves with some sun exposure. On examination, you notice scabbing and crusting with some silvery scale, and you are observant enough to notice small “pits” in his nails. What would account for these findings?

A) Eczema
B) Pityriasis rosea
C) Psoriasis
D) Tinea infection
C) Psoriasis

This is a classic presentation of plaque psoriasis. Eczema is usually over the flexor
surfaces and does not scale, whereas psoriasis affects the extensor surfaces. Pityriasis usually is
limited to the trunk and proximal extremities. Tinea has a much finer scale associated with it,
almost like powder, and is found in dark and moist areas.

Mrs. Anderson presents with an itchy rash which is raised and appears and disappears in
various locations. Each lesion lasts for many minutes. What most likely accounts for this
rash?

A) Insect bites
B) Urticaria, or hives
C) Psoriasis
D) Purpura
B) Urticaria, or hives

This is a typical case of urticaria. The most unusual aspect of this condition is that
the lesions “move” from place to place. This would be distinctly unusual for the other causes
listed.

Ms. Whiting is a 68 year old who comes in for her usual follow-up visit. You notice a few flat red and purple lesions, about 6 cm in diameter, on the ulnar aspect of her forearms but nowhere else. She doesn’t mention them. They are tender when you examine them. What should you do?

A) Conclude that these are lesions she has had for a long time.
B) Wait for her to mention them before asking further questions.
C) Ask how she acquired them.
D) Conduct the visit as usual for the patient.
C) Ask how she acquired them.

These are consistent with ecchymoses, or bruises. It is important to ask about antiplatelet medications such as aspirin, trauma history, and history of blood disorders in the patient and her family. Because of the different ages of the bruises and the isolation of them to
the ulnar forearms, these may be a result of abuse or other violence. It is your duty to investigate the cause of these lesions.

A middle-aged man comes in because he has noticed multiple small, blood-red, raised
lesions over his anterior chest and abdomen for the past several months. They are not painful
and he has not noted any bleeding or bruising. He is concerned this may be consistent with a dangerous condition. What should you do?

A) Reassure him that there is nothing to worry about.
B) Do laboratory work to check for platelet problems.
C) Obtain an extensive history regarding blood problems and bleeding disorders.
D) Do a skin biopsy in the office.
A) Reassure him that there is nothing to worry about.

These represent cherry angiomas, which are very common, benign lesions. Further
workup such as laboratory work, skin biopsy, or even further questions are not necessary at this
time. It would be wise to ask the patient to report any changes in any of his skin lesions, and
tell him that you would need to see him at that time.

A 38-year-old accountant comes to your clinic for evaluation of a headache. The throbbing
sensation is located in the right temporal region and is an 8 on a scale of 1 to 10. It started a few
hours ago, and she has noted nausea with sensitivity to light; she has had headaches like this in
the past, usually less than one per week, but not as severe. She does not know of any inciting factors. There has been no change in the frequency of her headaches. She usually takes an over-the-counter analgesic and this results in resolution of the headache. Based on this description, what is the most likely diagnosis of the type of headache?

A) Tension
B) Migraine
C) Cluster
D) Analgesic rebound
B) Migraine

This is a description of a common migraine (no aura). Distinctive features of a migraine include phonophobia and photophobia, nausea, resolution with sleep, and unilateral
distribution. Only some of these features may be present.

A 29-year-old computer programmer comes to your office for evaluation of a headache. The
tightening sensation is located all over the head and is of moderate intensity. It used to last
minutes, but this time it has lasted for 5 days. He denies photophobia and nausea. He spends
several hours each day at a computer monitor/keyboard. He has tried over-the-counter
medication; it has dulled the pain but not taken it away. Based on this description, what is your
most likely diagnosis?

A) Tension
B) Migraine
C) Cluster
D) Analgesic rebound
A) Tension

This is a description of a typical tension headache.

Which of the following is a symptom involving the eye?

A) Scotomas
B) Tinnitus
C) Dysphagia
D) Rhinorrhea
A) Scotomas

Scotomas are specks in the vision or areas where the patient cannot see; therefore, this is a common/concerning symptom of the eye.

A 49-year-old administrative assistant comes to your office for evaluation of dizziness. You
elicit the information that the dizziness is a spinning sensation of sudden onset, worse with head
position changes. The episodes last a few seconds and then go away, and they are accompanied
by intense nausea. She has vomited one time. She denies tinnitus. You perform a physical
examination of the head and neck and note that the patient’s hearing is intact to Weber and Rinne and that there is nystagmus. Her gait is normal. Based on this description, what is the most likely
diagnosis?

A) Benign positional vertigo
B) Vestibular neuronitis
C) Ménière’s disease
D) Acoustic neuroma
A) Benign positional vertigo

This is a classic description of benign positional vertigo. The vertigo is episodic,
lasting a few seconds to minutes, instead of continuous as in vestibular neuronitis. Also, there is
no tinnitus or sensorineural hearing loss as occurs in Ménière’s disease and acoustic neuroma.

A 55-year-old bank teller comes to your office for persistent episodes of dizziness. The first
episode started suddenly and lasted 3 to 4 hours. He experienced a lot of nausea with vomiting; the episode resolved spontaneously. He has had five episodes in the past 1.5 weeks. He does note some tinnitus that comes and goes. Upon physical examination, you note that he has a normal gait. The Weber localizes to the right side and the air conduction is equal to the bone conduction in the right ear. Nystagmus is present. Based on this description, what is the most likely diagnosis?

A) Benign positional vertigo
B) Vestibular neuronitis
C) Ménière’s disease
D) Acoustic neuroma
C) Ménière’s disease

Ménière’s disease is characterized by sudden onset of vertiginous episodes that last several hours to a day or more, then spontaneously resolve; the episodes then recur. On physical examination, sensorineural hearing loss is present. The patient does complain of tinnitus.

A 73-year-old nurse comes to your office for evaluation of new onset of tremors. She is not
on any medications and does not take herbs or supplements. She has no chronic medical conditions. She does not smoke or drink alcohol. She walks into the examination room with slow
movements and shuffling steps. She has decreased facial mobility and a blunt expression, without any changes in hair distribution on her face. Based on this description, what is the most likely reason for the patient’s symptoms?

A) Cushing’s syndrome
B) Nephrotic syndrome
C) Myxedema
D) Parkinson’s disease
D) Parkinson’s disease

This is a typical description for a patient with Parkinson’s disease. Facial mobility is
decreased, which results in a blunt expression—a “masked” appearance.

A 29-year-old physical therapist presents for evaluation of an eyelid problem. On
observation, the right eyeball appears to be protruding forward. Based on this description, what
is the most likely diagnosis?

A) Ptosis
B) Exophthalmos
C) Ectropion
D) Epicanthus
B) Exophthalmos

Feedback: Exophthalmos is the condition when the eyeball protrudes forward. If it is bilateral, it suggests the presence of Graves’ disease. If it is unilateral, it could still be caused by Graves’ disease. Alternatively, it could be caused by a tumor or inflammation in the orbit.

A 12-year-old presents to the clinic with his father for evaluation of a painful lump in the left
eye. It started this morning. He denies any trauma or injury. There is no visual disturbance. Upon
physical examination, there is a red raised area at the margin of the eyelid that is tender to
palpation; no tearing occurs with palpation of the lesion. Based on this description, what is the
most likely diagnosis?

A) Dacryocystitis
B) Chalazion
C) Hordeolum
D) Xanthelasma
C) Hordeolum

A hordeolum, or sty, is a painful, tender, erythematous infection in a gland at the margin of the eyelid.

A 15-year-old high school sophomore presents to the emergency room with his mother for evaluation of an area of blood in the left eye. He denies trauma or injury but has been coughing
forcefully with a recent cold. He denies visual disturbances, eye pain, or discharge from the eye.
On physical examination, the pupils are equal, round, and reactive to light, with a visual acuity of 20/20 in each eye and 20/20 bilaterally. There is a homogeneous, sharply demarcated area at the
lateral aspect of the base of the left eye. The cornea is clear. Based on this description, what is
the most likely diagnosis?

A) Conjunctivitis
B) Acute iritis
C) Corneal abrasion
D) Subconjunctival hemorrhage
D) Subconjunctival hemorrhage

A subconjunctival hemorrhage is a leakage of blood outside of the vessels, which
produces a homogenous, sharply demarcated bright red area; it fades over several days, turning yellow, then disappears. There is no associated eye pain, ocular discharge, or changes in visual
acuity; the cornea is clear. Many times it is associated with severe cough, choking, or vomiting,
which increase venous pressure.

Which of the following is a “red flag” regarding patients presenting with headache?

A) Unilateral headache
B) Pain over the sinuses
C) Age over 50
D) Phonophobia and photophobia
C) Age over 50

A unilateral headache is often seen with migraines and may commonly be
accompanied by phonophobia and photophobia. Pain over the sinuses from sinus congestion may also be unilateral and produce pain. Migraine and sinus headaches are common and
generally benign. A new severe headache in someone over 50 can be associated with more serious etiologies for headache.

A sudden, painless unilateral vision loss may be caused by which of the following?

A) Retinal detachment
B) Corneal ulcer
C) Acute glaucoma
D) Uveitis
A) Retinal detachment

Corneal ulcer, acute glaucoma, and uveitis are almost always accompanied by pain. Retinal detachment is generally painless, as is chronic glaucoma.

Sudden, painful unilateral loss of vision may be caused by which of the following
conditions?

A) Vitreous hemorrhage
B) Central retinal artery occlusion
C) Macular degeneration
D) Optic neuritis
D) Optic neuritis

In multiple sclerosis, sudden painful loss of vision may accompany optic neuritis. The other conditions are usually painless.

Diplopia, which is present with one eye covered, can be caused by which of the following
problems?

A) Weakness of CN III
B) Weakness of CN IV
C) A lesion of the brainstem
D) An irregularity in the cornea or lens
D) An irregularity in the cornea or lens

Double vision in one eye alone points to a problem in “processing” the light rays of an incoming image. The other causes of diplopia result in a misalignment of the two eyes.

A patient complains of epistaxis. Which other cause should be considered?

A) Intracranial hemorrhage
B) Hematemesis
C) Intestinal hemorrhage
D) Hematoma of the nasal septum
B) Hematemesis

Although the source of epistaxis may seem obvious, other bleeding locations should be on the differential. Hematemesis can mimic this and cause delay in life-saving therapies if
not considered. Intracranial hemorrhage and septal hematoma are instances of contained bleeding. Intestinal hemorrhage may cause hematemesis if there is obstruction distal to the bleeding, but this is unlikely.

Glaucoma is the leading cause of blindness in African-Americans and the second leading
cause of blindness overall. What features would be noted on funduscopic examination?

A) Increased cup-to-disc ratio
B) AV nicking
C) Cotton wool spots
D) Microaneurysms
A) Increased cup-to-disc ratio

It is important to screen for glaucoma on funduscopic examination. The cup and disc are among the easiest features to find. AV nicking and cotton wool spots are seen in
hypertension. Microaneurysms are seen in diabetes.

Very sensitive methods for detecting hearing loss include which of the following?

A) The whisper test
B) The finger rub test
C) The tuning fork test
D) Audiometric testing
D) Audiometric testing

While it is important to screen for hearing complaints with methods available to
you, it should be realized that some physical examination techniques are limited.

Which area of the fundus is the central focal point for incoming images?

A) The fovea
B) The macula
C) The optic disk
D) The physiologic cup
A) The fovea

The fovea is the area of the retina which is responsible for central vision. It is
surrounded by the macula, which is responsible for more peripheral vision. The optic disc and
physiologic cup are where the optic nerve enters the eye.

A light is pointed at a patient’s pupil, which contracts. It is also noted that the other pupil
contracts as well, though it is not exposed to bright light. Which of the following terms describes this latter phenomenon?

A) Direct reaction
B) Consensual reaction
C) Near reaction
D) Accommodation
The constriction of the contralateral pupil is called the consensual reaction. The response of the ipsilateral eye is the direct response. The dilation of the pupil when focusing on
a close object is the near reaction. Accommodation is the changing of the shape of the lens to
sharply focus on an object.

A patient is assigned a visual acuity of 20/100 in her left eye. Which of the following is true?

A) She obtains a 20% correct score at 100 feet.
B) She can accurately name 20% of the letters at 20 feet.
C) She can see at 20 feet what a normal person could see at 100 feet.
D) She can see at 100 feet what a normal person could see at 20 feet.
C) She can see at 20 feet what a normal person could see at 100 feet.

The denominator of an acuity score represents the line on the chart the patient can read. In the example above, the patient could read the larger letters corresponding with what a normal person could see at 100 feet.

On visual confrontation testing, a stroke patient is unable to see your fingers on his entire
right side with either eye covered. Which of the following terms would describe this finding?

A) Bitemporal hemianopsia
B) Right temporal hemianopsia
C) Right homonymous hemianopsia
D) Binasal hemianopsia
C) Right homonymous hemianopsia

Because the right visual field in both eyes is affected, this is a right homonymous
hemianopsia. A bitemporal hemianopsia refers to loss of both lateral visual fields. A right
temporal hemianopsia is unilateral and binasal hemianopsia is the loss of the nasal visual fields
bilaterally.

You note that a patient has anisocoria on examination. Pathologic causes of this include
which of the following?

A) Horner’s syndrome
B) Benign anisocoria
C) Differing light intensities for each eye
D) Eye prosthesis
A) Horner’s syndrome

A patient presents with ear pain. She is an avid swimmer. The history includes pain and
drainage from the left ear. On examination, she has pain when the ear is manipulated, including
manipulation of the tragus. The canal is narrowed and erythematous, with some white debris in the canal. The rest of the examination is normal. What diagnosis would you assign this
patient?

A) Otitis media
B) External otitis
C) Perforation of the tympanum
D) Cholesteatoma
B) External otitis

A patient with hearing loss by whisper test is further examined with a tuning fork, using the
Weber and Rinne maneuvers. The abnormal results are as follows: bone conduction is greater
than air on the left, and the patient hears the sound of the tuning fork better on the left. Which of the following is most likely?

A) Otosclerosis of the left ear
B) Exposure to chronic loud noise of the right ear
C) Otitis media of the right ear
D) Perforation of the right eardrum
A) Otosclerosis of the left ear

The above pattern is consistent with a conductive loss on the left side. Causes
would include: foreign body, otitis media, perforation, and otosclerosis of the involved side.

A young man is concerned about a hard mass he has just noticed in the midline of his palate. On examination, it is indeed hard and in the midline. There are no mucosal abnormalities associated with this lesion. He is experiencing no other symptoms. What will
you tell him is the most likely diagnosis?

A) Leukoplakia
B) Torus palatinus
C) Thrush (candidiasis)
D) Kaposi’s sarcoma
B) Torus palatinus

Torus palatinus is relatively common and benign but can go unnoticed by the patient for many years. The appearance of a bony mass can be concerning.

A young woman undergoes cranial nerve testing. On touching the soft palate, her uvula
deviates to the left. Which of the following is likely?

A) CN IX lesion on the left
B) CN IX lesion on the right
C) CN X lesion on the left
D) CN X lesion on the right
D) CN X lesion on the right

The failure of the right side of the palate to rise denotes a problem with the right
10th cranial nerve. The uvula deviates toward the properly functioning side.

A college student presents with a sore throat, fever, and fatigue for several days. You notice exudates on her enlarged tonsils. You do a careful lymphatic examination and notice some scattered small, mobile lymph nodes just behind her sternocleidomastoid muscles
bilaterally. What group of nodes is this?

A) Submandibular
B) Tonsillar
C) Occipital
D) Posterior cervical
D) Posterior cervical

The group of nodes posterior to the sternocleidomastoid muscle is the posterior cervical chain. These are common in mononucleosis.

You feel a small mass that you think is a lymph node. It is mobile in both the up-and-down and side-to-side directions. Which of the following is most likely?

A) Cancer
B) Lymph node
C) Deep scar
D) Muscle
B) Lymph node

A useful maneuver for discerning lymph nodes from other masses in the neck is to check for their mobility in all directions. Many other masses are mobile in only two directions. Cancerous masses may also be “fixed,” or immobile.

You are conducting a pupillary examination on a 34-year-old man. You note that both pupils dilate slightly. Both are noted to constrict briskly when the light is placed on the right
eye. What is the most likely problem?

A) Optic nerve damage on the right
B) Optic nerve damage on the left
C) Efferent nerve damage on the right
D) Efferent nerve damage on the left
B) Optic nerve damage on the left

Because both pupils can constrict, efferent nerve damage is unlikely. When the light is placed on the left eye, neither a direct nor a consensual response is seen. This indicates that the left eye is not perceiving incoming light.

A 21-year-old college senior presents to your clinic, complaining of shortness of breath and a nonproductive nocturnal cough. She states she used to feel this way only with extreme
exercise, but lately she has felt this way continuously. She denies any other upper respiratory
symptoms, chest pain, gastrointestinal symptoms, or urinary tract symptoms. Her past medical history is significant only for seasonal allergies, for which she takes a nasal steroid spray but is otherwise on no other medications. She has had no surgeries. Her mother has allergies and eczema and her father has high blood pressure. She is an only child. She denies smoking and
illegal drug use but drinks three to four alcoholic beverages per weekend. She is a junior in
finance at a local university and she has recently started a job as a bartender in town. On
examination she is in no acute distress and her temperature is 98.6. Her blood pressure is 120/80,
her pulse is 80, and her respirations are 20. Her head, eyes, ears, nose, and throat examinations are essentially normal. Inspection of her anterior and posterior chest shows no abnormalities. On auscultation of her chest, there is decreased air movement and a high-pitched whistling on expiration in all lobes. Percussion reveals resonant lungs.
Which disorder of the thorax or lung does this best describe?

A) Spontaneous pneumothorax
B) Chronic obstructive pulmonary disease (COPD)
C) Asthma
D) Pneumonia
C) Asthma

Asthma causes shortness of breath and a nocturnal cough. It is often associated with
a history of allergies and can be made worse by exercise or irritants such as smoke in a bar. On
auscultation there can be normal to decreased air movement. Wheezing is heard on expiration
and sometimes inspiration. The duration of wheezing in expiration usually correlates with
severity of illness, so it is important to document this length (e.g., wheezes heard halfway through exhalation). Realize that in severe asthma, wheezes may not be heard because of the
lack of air movement.

A 47-year-old receptionist comes to your office, complaining of fever, shortness of breath,
and a productive cough with golden sputum. She says she had a cold last week and her
symptoms have only gotten worse, despite using over-the-counter cold remedies. She denies any
weight gain, weight loss, or cardiac or gastrointestinal symptoms. Her past medical history includes type 2 diabetes for 5 years and high cholesterol. She takes an oral medication for both diseases. She has had no surgeries. She denies tobacco, alcohol, or drug use. Her mother has diabetes and high blood pressure. Her father passed away from colon cancer. On examination you see a middle-aged woman appearing her stated age. She looks ill and her temperature is elevated, at 101. Her blood pressure and pulse are unremarkable. Her head, eyes, ears, nose, and throat examinations are unremarkable except for edema of the nasal turbinates. On auscultation she has decreased air movement, and coarse crackles are heard over the left lower lobe. There is dullness on percussion, increased fremitus during palpation, and egophony and whispered pectoriloquy on auscultation.
What disorder of the thorax or lung best describes her symptoms?

A) Spontaneous pneumothorax
B) Chronic obstructive pulmonary disease (COPD)
C) Asthma
D) Pneumonia
D) Pneumonia

Pneumonia is usually associated with dyspnea, cough, and fever. On auscultation
there can be coarse or fine crackles heard over the affected lobe. Percussion over the affected area is dull and there is often an increase in fremitus. Egophony and pectoriloquy are heard because of increased transmission of high-pitched components of sounds. These higher frequencies are usually filtered out by the multiple air-filled chambers of the alveoli.

A 17-year-old high school senior presents to your clinic in acute respiratory distress. Between shallow breaths he states he was at home finishing his homework when he suddenly began having right-sided chest pain and severe shortness of breath. He denies any recent traumas or illnesses. His past medical history is unremarkable. He doesn’t smoke but drinks several beers on the weekend. He has tried marijuana several times but denies any other illegal drugs. He is an honors student and is on the basketball team. His parents are both in good health. He denies any recent weight gain, weight loss, fever, or night sweats. On examination you see a tall, thin young man in obvious distress. He is diaphoretic and is breathing at a rate of 35 breaths per minute. On
auscultation you hear no breath sounds on the right side of his superior chest wall. On percussion
he is hyperresonant over the right upper lobe. With palpation he has absent fremitus over the
right upper lobe.
What disorder of the thorax or lung best describes his symptoms?

A) Spontaneous pneumothorax
B) Chronic obstructive pulmonary disease (COPD)
C) Asthma
D) Pneumonia
A) Spontaneous pneumothorax

Spontaneous pneumothorax occurs suddenly, causing severe dyspnea and chest pain on the affected side. It is more common in thin young males. On auscultation of the affected side there will be no breath sounds and on percussion there is hyperresonance or tympany. There will
be an absence of fremitus to palpation.

A 62-year-old construction worker presents to your clinic, complaining of almost a year of
chronic cough and occasional shortness of breath. Although he has had worsening of symptoms
occasionally with a cold, his symptoms have stayed about the same. The cough has occasional
mucous drainage but never any blood. He denies any chest pain. He has had no weight gain,
weight loss, fever, or night sweats. His past medical history is significant for high blood pressure
and arthritis. He has smoked two packs a day for the past 45 years. He drinks occasionally but
denies any illegal drug use. He is married and has two children. He denies any foreign travel. His
father died of a heart attack and his mother died of Alzheimer’s disease. On examination you see
a man looking slightly older than his stated age. His blood pressure is 130/80 and his pulse is 88.
He is breathing comfortably with respirations of 12. His head, eyes, ears, nose, and throat
examinations are unremarkable. His cardiac examination is normal. On examination of his chest,
the diameter seems enlarged. Breath sounds are decreased throughout all lobes. Rhonchi are
heard over all lung fields. There is no area of dullness and no increased or decreased fremitus.
What thorax or lung disorder is most likely causing his symptoms?
A) Spontaneous pneumothorax
B) Chronic obstructive pulmonary disease (COPD)
C) Asthma
D) Pneumonia
B) Chronic obstructive pulmonary disease (COPD)

This disorder is insidious in onset and generally affects the older population with a smoking history. The diameter of the chest is often enlarged like a barrel. Percussing the chest elicits hyperresonance, and during auscultation there are often distant breath sounds. Coarse
breath sounds of rhonchi are also often heard. It is important to quantify this patient’s exercise
capacity because it may affect his employment and also allows you to follow for progression of
his disease. You must offer smoking cessation as an option.

A 36-year-old teacher presents to your clinic, complaining of sharp, knifelike pain on the left
side of her chest for the last 2 days. Breathing and lying down make the pain worse, while sitting
forward helps her pain. Tylenol and ibuprofen have not helped. Her pain does not radiate to any
other area. She denies any upper respiratory or gastrointestinal symptoms. Her past medical
history consists of systemic lupus. She is divorced and has one child. She denies any tobacco,
alcohol, or drug use. Her mother has hypothyroidism and her father has high blood pressure. On
examination you find her to be distressed, leaning over and holding her left arm and hand to her left chest. Her blood pressure is 130/70, her respirations are 12, and her pulse is 90. On
auscultation her lung fields have normal breath sounds with no rhonchi, wheezes, or crackles.
Percussion and palpation are unremarkable. Auscultation of the heart has an S1 and S2 with no S3
or S4. A scratching noise is heard at the lower left sternal border, coincident with systole; leaning
forward relieves some of her pain. She is nontender with palpation of the chest wall.
What disorder of the chest best describes this disorder?

A) Angina pectoris
B) Pericarditis
C) Dissecting aortic aneurysm
D) Pleural pain
B) Pericarditis

The pain from pericarditis is usually sharp and knifelike and is located over the left side of the chest. Change of position, breathing, and coughing often make the pain worse,
whereas leaning forward improves the pain. Pericarditis is often seen in rheumatologic diseases such as systemic lupus and in patients with chronic kidney disease. Patients also experience this
after a myocardial infarction.

A 68-year-old retired postman presents to your clinic, complaining of dull, intermittent
left-sided chest pain over the last few weeks. The pain occurs after he mows his lawn or chops
wood. He says that the pain radiates to the left side of his jaw but nowhere else. He has felt
light-headed and nauseated with the pain but has had no other symptoms. He states when he sits
down for several minutes the pain goes away. Ibuprofen, Tylenol, and antacids have not improved his symptoms. He reports no recent weight gain, weight loss, fever, or night sweats. He has a past medical history of high blood pressure and arthritis. He quit smoking 10 years ago after smoking one pack a day for 40 years. He denies any recent alcohol use and reports no drug
use. He is married and has two healthy children. His mother died of breast cancer and his father
died of a stroke. His younger brother has had bypass surgery. On examination you find him
healthy-appearing and breathing comfortably. His blood pressure is 140/90 and he has a pulse of

  1. His head, eyes, ears, nose, and throat examinations are unremarkable. His lungs have normal breath sounds and there are no abnormalities with percussion and palpation of the chest. His heart has a normal S1 and S2 and no S3 or S4. Further workup is pending.
    Which disorder of the chest best describes these symptoms?

A) Angina pectoris
B) Pericarditis
C) Dissecting aortic aneurysm
D) Pleural pain
A) Angina pectoris

Angina causes dull chest pain felt in the retrosternal area or anterior chest. It often radiates to the shoulders, arms, neck, and jaw. It is associated with shortness of breath, nausea, and sweating. The pain is generally relieved by rest or medication after several minutes. This patient needs to be admitted to the hospital for further workup for his accelerating symptoms.

A 75-year-old retired teacher presents to your clinic, complaining of severe, unrelenting
anterior chest pain radiating to her back. She describes it as if someone is “ripping out her heart.”
It began less than an hour ago. She states she is feeling very nauseated and may pass out. She
denies any trauma or recent illnesses. She states she has never had pain like this before. Nothing
seems to make the pain better or worse. Her medical history consists of difficult-to-control
hypertension and coronary artery disease requiring two stents in the past. She is a widow. She denies any alcohol, tobacco, or illegal drug use. Her mother died of a stroke and her father died of a heart attack. She has one younger brother who has had bypass surgery. On examination you see an elderly female in a great deal of distress. She is lying on the table, curled up, holding her left and right arms against her chest and is restless, trying to find a comfortable position. Her
blood pressure is 180/110 in the right arm and 130/60 in the left arm, and her pulse is 120. Her
right carotid pulse is bounding but the left carotid pulse is weak. She is afebrile but her respirations are 24 times a minute. On auscultation her lungs are clear and her cardiac
examination is unremarkable. You call EMS and have her taken to the hospital’s ER for further
evaluation.
What disorder of the chest best describes her symptoms?

A) Angina pectoris
B) Pericarditis
C) Dissecting aortic aneurysm
D) Pleural pain
C) Dissecting aortic aneurysm

A dissecting aortic aneurysm is associated with a ripping or tearing sensation that
radiates to the neck, back, or abdomen. Because blood supply to the brain and extremities is
disrupted, syncope and paraplegia or hemiplegia can occur. Blood pressure will usually be different between the two arms, and the carotid pulses often show an asymmetry. This is because the aneurysm decreases flow distally and causes inequality of flow between sides.

A 25-year-old accountant presents to your clinic, complaining of intermittent lower right-sided chest pain for several days. He describes it as knifelike and states it only lasts for 3 to 5 seconds, taking his breath away. He states he feels like he has to breathe shallowly to keep it from recurring. The only thing that makes it better is lying quietly on his right side. It is much worse when he takes a deep breath. He has taken some Tylenol and put a heating pad on his side but neither has helped. He remembers that 2 weeks ago he had an upper respiratory infection
with a severe hacking cough. He denies any recent trauma. His past medical history is unremarkable. His parents and siblings are in good health. He has recently married, and his wife has a baby due in 2 months. He denies any smoking or illegal drug use. He drinks two to three beers once a month. He states that he eats a healthy diet and runs regularly, but not since his recent illness. He denies any cardiac, gastrointestinal, or musculoskeletal symptoms. On
examination he is lying on his right side but appears quite comfortable. His temperature, blood pressure, pulse, and respirations are unremarkable. His chest has normal breath sounds on auscultation. Percussion of the chest is unremarkable. During palpation the ribs are nontender.
What disorder of the chest best describes his symptoms?

A) Pericarditis
B) Chest wall pain
C) Pleural pain
D) Angina pectoralis
C) Pleural pain

This pain is sharp and knifelike and occurs over the affected area of pleura.
Breathing deeply usually makes the pain worse, whereas lying quietly on the affected side makes
the pain better. Pleurisy often occurs from inflammation due to an infection, neoplasm, or
autoimmune disease.

A 60-year-old baker presents to your clinic, complaining of increasing shortness of breath
and nonproductive cough over the last month. She feels like she can’t do as much activity as she
used to do without becoming tired. She even has to sleep upright in her recliner at night to be able to breathe comfortably. She denies any chest pain, nausea, or sweating. Her past medical
history is significant for high blood pressure and coronary artery disease. She had a hysterectomy
in her 40s for heavy vaginal bleeding. She is married and is retiring from the local bakery soon. She denies any tobacco, alcohol, or drug use. Her mother died of a stroke and her father died from prostate cancer. She denies any recent upper respiratory illness, and she has had no other symptoms. On examination she is in no acute distress. Her blood pressure is 160/100 and her pulse is 100. She is afebrile and her respiratory rate is 16. With auscultation she has distant air sounds and she has late inspiratory crackles in both lower lobes. On cardiac examination the S1 and S2 are distant and an S3 is heard over the apex.
What disorder of the chest best describes her symptoms?

A) Pneumonia
B) Chronic obstructive pulmonary disease (COPD)
C) Pleural pain
D) Left-sided heart failure
D) Left-sided heart failure

In left-sided heart failure, fluid starts “backing up” into the lungs because the heart is unable to handle the volume. The excess fluid collects in the dependent areas, causing crackles
in the bases of the lower lobes. Sitting up allows patients to breathe easier. The two main causes
are chronic high blood pressure and coronary artery disease, which lead to myocardial ischemia and decreased contractility of the heart.

A grandmother brings her 13-year-old grandson to you for evaluation. She noticed last
week when he took off his shirt that his breastbone seemed collapsed. He seems embarrassed and tells you that it has been that way for quite awhile. He states he has no symptoms from it and he
just tries not to take off his shirt in front of anyone. He denies any shortness of breath, chest pain,
or lightheadedness on exertion. His past medical history is unremarkable. He is in sixth grade
and just moved in with his grandmother after his father was deployed to the Middle East. His
mother died several years ago in a car accident. He states that he does not smoke and has never
touched alcohol. On examination you see a teenage boy appearing his stated age. On visual
examination of his chest you see that the lower portion of the sternum is depressed. Auscultation
of the lungs and heart are unremarkable.

What disorder of the thorax best describes your findings?
A) Barrel chest
B) Funnel chest (pectus excavatum)
C) Pigeon chest (pectus carinatum)
D) Thoracic kyphoscoliosis
B) Funnel chest (pectus excavatum)

Funnel chest is caused by a depression in the lower portion of the sternum. If severe
enough there can be compression of the heart and great vessels, leading to murmurs on
auscultation. This is usually only a cosmetic problem, but corrective surgeries can be
performed if necessary.

Which of the following anatomic landmark associations is correct?

A) 2nd intercostal space for needle insertion in tension pneumothorax
B) T6 for lower margin of endotracheal tube
C) Sternal angle marks the 4th rib
D) 5th intercostal space for chest tube insertion
A) 2nd intercostal space for needle insertion in tension pneumothorax

The 2nd intercostal space is indeed the correct location for insertion of a needle in tension pneumothorax. The other answers are incorrect.

A 55-year-old smoker complains of chest pain and gestures with a closed fist over her sternum to describe it. Which of the following diagnoses should you consider because of her gesture?

A) Bronchitis
B) Costochondritis
C) Pericarditis
D) Angina pectoris
D) Angina pectoris

The clenched fist of Levine’s sign, while not completely specific for ischemic pain, should definitely cause you to consider this etiology.

A 62-year-old smoker complains of “coughing up small amounts of blood,” so you consider hemoptysis. Which of the following should you also consider?

A) Intestinal bleeding
B) Hematoma of the nasal septum
C) Epistaxis
D) Bruising of the tongue
C) Epistaxis

When you suspect hemoptysis, you must consider other etiologies for bleeding.
Commonly, epistaxis can mimic this as well as bleeding from the gastrointestinal tract. The other
answers, although they involve bleeding, are contained or distant from the pharynx.

Which of the following occurs in respiratory distress?

A) Speaking in sentences of 10-20 words
B) Skin between the ribs moves inward with inspiration
C) Neck muscles are relaxed
D) Patient torso leans posteriorly
B) Skin between the ribs moves inward with inspiration

This description is consistent with retractions that occur with respiratory distress. Other features include speaking in short sentences, use of accessory muscles, leaning forward
to gain mechanical advantage for the diaphragm, and pursed lip breathing, in which the patient
exhales against his lips, which are pressed together.

Which of the following is consistent with good percussion technique?

A) Allow all of the fingers to touch the chest while performing percussion.
B) Maintain a stiff wrist and hand.
C) Leave the plexor finger on the pleximeter after each strike.
D) Strike the pleximeter over the distal interphalangeal joint.
D) Strike the pleximeter over the distal interphalangeal joint

Percussion takes practice to master. Most struggle initially with keeping the wrist and hand relaxed. Other challenges include removing the plexor quickly and keeping the other
fingers off the chest wall.

Which of the following percussion notes would you obtain over the gastric bubble?

A) Resonance
B) Tympany
C) Hyperresonance
D) Flatness
B) Tympany

The gastric bubble produces one of the longest percussion notes. A patient with
COPD may have hyperresonance over his chest, while a normal person would have resonance.
Dullness is heard over a normal liver, and flatness is heard if one percusses a large muscle.

Which of the following conditions would produce a hyperresonant percussion note?

A) Large pneumothorax
B) Lobar pneumonia
C) Pleural effusion
D) Empyema
A) Large pneumothorax

There is a great deal of free air in the chest with a large pneumothorax, which
produces a hyperresonant note. The other three conditions produce dullness by dampening the
percussion note with fluid.

Which lung sound possesses the characteristics of being louder and higher in pitch, with a short silence between inspiration and expiration and with expiration being longer than inspiration?

A) Bronchovesicular
B) Vesicular
C) Bronchial
D) Tracheal
C) Bronchial

A patient complains of shortness of breath for the past few days. On examination, you note late inspiratory crackles in the lower third of the chest that were not present a week ago.
What is the most likely explanation for these?

A) Asthma
B) COPD
C) Bronchiectasis
D) Heart failure
D) Heart failure

The timing of crackles within inspiration provides important clues. These late
inspiratory crackles that appeared suddenly would be most consistent with heart failure. COPD
and asthma usually produce early inspiratory crackles. Bronchiectasis, as seen in cystic
fibrosis, classically produces mid-inspiratory crackles, but this is not always reliable.

When crackles, wheezes, or rhonchi clear with a cough, which of the following is a likely
etiology?

A) Bronchitis
B) Simple asthma
C) Cystic fibrosis
D) Heart failure
A) Bronchitis

Adventitious sounds that clear with cough are usually consistent with bronchitis or
atelectasis. The other conditions would not be associated with findings that cleared with a cough.

A patient with longstanding COPD was told by another practitioner that his liver was enlarged and this needed to be assessed. Which of the following would be reasonable to do next?

A) Percuss the lower border of the liver
B) Measure the span of the liver
C) Order a hepatitis panel
D) Obtain an ultrasound of the liver
B) Measure the span of the liver

In this patient, measuring the span of the liver saved the patient an involved workup, because it was normal. His history of COPD is consistent with flattening of the diaphragms, which pushed the liver edge down while the actual size of the liver remained the same. Percussing the lower border of the liver alone caused this referral, because it was assumed that
the liver was enlarged.

You are at your family reunion playing football when your uncle takes a hit to his right
lateral thorax and is in pain. He asks you if you think he has a rib fracture. You are in a very
remote area. What would your next step be?

A) Call a medevac helicopter
B) Drive him to the city (4 hours away)
C) Press on his sternum and spine simultaneously
D) Examine him for tenderness over the injured area
C) Press on his sternum and spine simultaneously

The area involved in the injury will of course be tender. If you press in an area
remote to the injury, but over the same bone which may be involved, you can produce tenderness
at the site of injury. This would indicate that there may be a fracture at the lateral ribs.

You are performing a thorough cardiac examination. Which of the following chambers of
the heart can you assess by palpation?

A) Left atrium
B) Right atrium
C) Right ventricle
D) Sinus node
C) Right ventricle

The right ventricle occupies most of the anterior cardiac surface and is easily
accessible to palpation.

What is responsible for the inspiratory splitting of S2?

A) Closure of aortic, then pulmonic valves
B) Closure of mitral, then tricuspid valves
C) Closure of aortic, then tricuspid valves
D) Closure of mitral, then pulmonic valves
A) Closure of aortic, then pulmonic valves

A 25-year-old optical technician comes to your clinic for evaluation of fatigue. As part of your physical examination, you listen to her heart and hear a murmur only at the cardiac apex.
Which valve is most likely to be involved, based on the location of the murmur?

A) Mitral
B) Tricuspid
C) Aortic
D) Pulmonic
A) Mitral

Mitral valve sounds are usually heard best at and around the cardiac apex.

A 58-year-old teacher presents to your clinic with a complaint of breathlessness with
activity. The patient has no chronic conditions and does not take any medications, herbs, or
supplements. Which of the following symptoms is appropriate to ask about in the cardiovascular
review of systems?

A) Abdominal pain
B) Orthopnea
C) Hematochezia
D) Tenesmus
B) Orthopnea

You are screening people at the mall as part of a health fair. The first person who comes for
screening has a blood pressure of 132/85. How would you categorize this?

A) Normal
B) Prehypertension
C) Stage 1 hypertension
D) Stage 2 hypertension
B) Prehypertension

Prehypertension is considered to be a systolic blood pressure from 120 to 139 and a diastolic BP from 80 to 89.

You are participating in a health fair and performing cholesterol screens. One person has a
cholesterol of 225. She is concerned about her risk for developing heart disease. Which of the following factors is used to estimate the 10-year risk of developing coronary heart disease?

A) Ethnicity
B) Alcohol intake
C) Gender
D) Asthma
C) Gender

Gender is used in the calculation of the 10-year risk for developing coronary heart disease, because men have a higher risk than women.

You are evaluating a 40-year-old banker for coronary heart disease risk factors. He has a history of hypertension, which is well-controlled on his current medications. He does not smoke;
he does 45 minutes of aerobic exercise five times weekly. You are calculating his 10-year
coronary heart disease risk. Which of the following conditions is considered to be a coronary
heart disease risk equivalent?

A) Hypertension
B) Peripheral arterial disease
C) Systemic lupus erythematosus
D) Chronic obstructive pulmonary disease (COPD)
B) Peripheral arterial disease

Peripheral arterial disease is considered to be a coronary heart disease risk
equivalent, as are abdominal aortic aneurysm, carotid atherosclerotic disease, and diabetes
mellitus.

You are conducting a workshop on the measurement of jugular venous pulsation. As part of
your instruction, you tell the students to make sure that they can distinguish between the jugular venous pulsation and the carotid pulse. Which one of the following characteristics is typical of the carotid pulse?

A) Palpable
B) Soft, rapid, undulating quality
C) Pulsation eliminated by light pressure on the vessel
D) Level of pulsation changes with changes in position
A) Palpable

The carotid pulse is palpable; the jugular venous pulsation is rarely palpable. The
carotid upstroke is normally brisk, but it may be delayed and decreased as in aortic stenosis or
bounding as in aortic insufficiency.

A 68-year-old mechanic presents to the emergency room for shortness of breath. You are
concerned about a cardiac cause and measure his jugular venous pressure (JVP). It is elevated.
Which one of the following conditions is a potential cause of elevated JVP?

A) Left-sided heart failure
B) Mitral stenosis
C) Constrictive pericarditis
D) Aortic aneurysm
C) Constrictive pericarditis

One cause of increased jugular venous pressure is constrictive pericarditis. Others
include right-sided heart failure, tricuspid stenosis, and superior vena cava syndrome.

You are palpating the apical impulse in a patient with heart disease and find that the
amplitude is diffuse and increased. Which of the following conditions could be a potential cause
of an increase in the amplitude of the impulse?

A) Hypothyroidism
B) Aortic stenosis, with pressure overload of the left ventricle
C) Mitral stenosis, with volume overload of the left atrium
D) Cardiomyopathy
B) Aortic stenosis, with pressure overload of the left ventricle

You are performing a cardiac examination on a patient with shortness of breath and
palpitations. You listen to the heart with the patient sitting upright, then have him change to a
supine position, and finally have him turn onto his left side in the left lateral decubitus position.
Which of the following valvular defects is best heard in this position?

A) Aortic
B) Pulmonic
C) Mitral
D) Tricuspid
C) Mitral

The left lateral decubitus position brings the left ventricle closer to the chest wall,
allowing mitral valve murmurs to be better heard. If you do not listen to the heart in this position with both the diaphragm and bell in a quiet room, it is possible to miss significant
murmurs such as mitral stenosis.

You are concerned that a patient has an aortic regurgitation murmur. Which is the best position to accentuate the murmur?

A) Upright
B) Upright, but leaning forward
C) Supine
D) Left lateral decubitus
B) Upright, but leaning forward

Leaning forward slightly in the upright position brings the aortic valve and the left ventricular outflow tract closer to the chest wall, so it will be easier to hear the soft diastolic
decrescendo murmur of aortic insufficiency (regurgitation).

A 68-year-old retired waiter comes to your clinic for evaluation of fatigue. You perform a
cardiac examination and find that his pulse rate is less than 60. Which of the following conditions could be responsible for this heart rate?

A) Second-degree A-V block
B) Atrial flutter
C) Sinus arrhythmia
D) Atrial fibrillation
A) Second-degree A-V block

Atrial flutter
and atrial fibrillation do not cause bradycardia unless there is a significant accompanying block.
Sinus arrhythmia does not cause bradycardia and represents respiratory variation of the heart
rate.

Where is the point of maximal impulse (PMI) normally located?

A) In the left 5th intercostal space, 7 to 9 cm lateral to the sternum
B) In the left 5th intercostal space, 10 to 12 cm lateral to the sternum
C) In the left 5th intercostal space, in the anterior axillary line
D) In the left 5th intercostal space, in the midaxillary line
A) In the left 5th intercostal space, 7 to 9 cm lateral to the sternum

Which of the following events occurs at the start of diastole?

A) Closure of the tricuspid valve
B) Opening of the pulmonic valve
C) Closure of the aortic valve
D) Production of the first heart sound (S1)
C) Closure of the aortic valve

It is thought that the closure of the aortic valve produces the second heart sound (S2). Closure of the mitral valve is thought to produce the first heart sound (S1).

Which is true of a third heart sound (S3)?

A) It marks atrial contraction.
B) It reflects normal compliance of the left ventricle.
C) It is caused by rapid deceleration of blood against the ventricular wall.
D) It is not heard in atrial fibrillation.
It is caused by rapid deceleration of blood against the ventricular wall.

Which is true of splitting of the second heart sound?

A) It is best heard over the pulmonic area with the bell of the stethoscope.
B) It normally increases with exhalation.
C) It is best heard over the apex.
D) It does not vary with respiration.
A) It is best heard over the pulmonic area with the bell of the stethoscope.

Which of the following is true of jugular venous pressure (JVP) measurement?

A) It is measured with the patient at a 45-degree angle.
B) The vertical height of the blood column in centimeters, plus 5 cm, is the JVP.
C) A JVP below 9 cm is abnormal.
D) It is measured above the sternal notch.
B) The vertical height of the blood column in centimeters, plus 5 cm, is the JVP.

How much does cardiovascular risk increase for each increment of 20 mm Hg systolic and
10 mm Hg diastolic in blood pressure?

A) 25%
B) 50%
C) 75%
D) 100%
D) 100%

Each increase of BP by 20 systolic and 10 diastolic doubles the risk of cardiovascular disease.

In healthy adults over 20, how often should blood pressure, body mass index, waist circumference, and pulse be assessed, according to American Heart Association guidelines?

A) Every 6 months
B) Every year
C) Every 2 years
D) Every 5 years
C) Every 2 years

Which of the following is a clinical identifier of metabolic syndrome?

A) Waist circumference of 38 inches for a male
B) Waist circumference of 34 inches for a female
C) BP of 134/88 for a male
D) BP of 128/84 for a female
C) BP of 134/88 for a male

The physical examination criteria for identifying metabolic syndrome include a waist of 40 inches or greater for a male, a waist of 35 inches or greater for a female, and a blood pressure of 130/85 or greater. Other criteria include triglycerides greater than or equal to 150 mg/dL, fasting glucose greater than or equal to 110 mg/dL, and HDL less than 40 for men or less than 50 for women.

Mrs. Adams would like to begin an exercise program and was told to exercise as intensely as necessary to obtain a heart rate 60% or greater of her maximum heart rate. She is 52. What heart rate should she achieve?

A) 80
B) 100
C) 120
D) 140
B) 100

Maximum heart rate is calculated by subtracting the patient’s age from 220. For Mrs. Adams, 60% of this number is about 100. She must also be instructed in how to measure
her own pulse or have a device to do so.

In measuring the jugular venous pressure (JVP), which of the following is important?

A) Keep the patient’s torso at a 45-degree angle.
B) Measure the highest visible pressure, usually at end expiration.
C) Add the vertical height over the sternal notch to a 5-cm constant.
D) Realize that a total value of over 12 cm is abnormal.
B) Measure the highest visible pressure, usually at end expiration.

You find a bounding carotid pulse on a 62-year-old patient. Which murmur should you
search out?

A) Mitral valve prolapse
B) Pulmonic stenosis
C) Tricuspid insufficiency
D) Aortic insufficiency
D) Aortic insufficiency

To hear a soft murmur or bruit, which of the following may be necessary?

A) Asking the patient to hold her breath
B) Asking the patient in the next bed to turn down the TV
C) Checking your stethoscope for air leaks
D) All of the above
D) All of the above

Which of the following may be missed unless the patient is placed in the left lateral
decubitus position and auscultated with the bell?

A) Mitral stenosis murmur
B) Opening snap of the mitral valve
C) S3 and S4 gallops
D) All of the above
D) All of the above

How should you determine whether a murmur is systolic or diastolic?

A) Palpate the carotid pulse.
B) Palpate the radial pulse.
C) Judge the relative length of systole and diastole by auscultation.
D) Correlate the murmur with a bedside heart monitor.
A) Palpate the carotid pulse.

Which of the following correlates with a sustained, high-amplitude PMI?

A) Hyperthyroidism
B) Anemia
C) Fever
D) Hypertension
D) Hypertension

You are examining a patient with emphysema in exacerbation and are having difficulty hearing his heart sounds. What should you do to obtain a good examination?

A) Listen in the epigastrium.
B) Listen to the patient in the left lateral decubitus position.
C) Ask the patient to hold his breath for 30 seconds.
D) Listen posteriorly.
A) Listen in the epigastrium.

You are listening carefully for S2 splitting. Which of the following will help?

A) Using the diaphragm with light pressure over the 2nd right intercostal space
B) Using the bell with light pressure over the 2nd left intercostal space
C) Using the diaphragm with firm pressure over the apex
D) Using the bell with firm pressure over the lower left sternal border
B) Using the bell with light pressure over the 2nd left intercostal space

  1. Which of the following is true of a grade 4-intensity murmur?

A) It is moderately loud.
B) It can be heard with the stethoscope off the chest.
C) It can be heard with the stethoscope partially off the chest.
D) It is associated with a “thrill.”
D) It is associated with a “thrill.”

The grade 4 murmur is differentiated from those below it by the presence of a
palpable thrill. A murmur cannot be graded as a 4 unless this is present.

Which valve lesion typically produces a murmur of equal intensity throughout systole?

A) Aortic stenosis
B) Mitral insufficiency
C) Pulmonic stenosis
D) Aortic insufficiency
B) Mitral insufficiency

You notice a patient has a strong pulse and then a weak pulse. This pattern continues.
Which of the following is likely?

A) Emphysema
B) Asthma exacerbation
C) Severe left heart failure
D) Cardiac tamponade
C) Severe left heart failure

Suzanne is a 20-year-old college student who complains of chest pain. This is intermittent
and is located to the left of her sternum. There are no associated symptoms. On examination,
you hear a short, high-pitched sound in systole, followed by a murmur which increases in
intensity until S2. This is heard best over the apex. When she squats, this noise moves later in
systole along with the murmur. Which of the following is the most likely diagnosis?

A) Mitral stenosis
B) Mitral insufficiency
C) Mitral valve prolapse
D) Mitral valve papillary muscle ischemia
C) Mitral valve prolapse

A 57-year-old maintenance worker comes to your office for evaluation of pain in his legs.
He has smoked two packs per day since the age of 16, but he is otherwise healthy. You are
concerned that he may have peripheral vascular disease. Which of the following is part of common or concerning symptoms for the peripheral vascular system?

A) Intermittent claudication
B) Chest pressure with exertion
C) Shortness of breath
D) Knee pain
A) Intermittent claudication

Intermittent claudication is leg pain that occurs with walking and is relieved by rest.
It is a key symptom of peripheral vascular disease.

A 72-year-old teacher comes to your clinic for an annual examination. She is concerned
about her risk for peripheral vascular disease and states that there is a place in town that does
tests to let her know her if she has this or not. Which of the following disease processes is a risk
factor for peripheral vascular disease?

A) Gastroesophageal reflux disease
B) Coronary artery disease
C) Migraine headaches
D) Osteoarthritis
B) Coronary artery disease

Evidence of coronary artery disease implies that there is most likely disease in other
vessels; therefore, this is a risk factor for peripheral vascular disease.

A 68-year-old retired truck driver comes to your office for evaluation of swelling in his legs.
He is a smoker and has been taking medications to control his hypertension for the past 25 years.
You are concerned about his risk for peripheral vascular disease. Which of the following tests are
appropriate to order to initially evaluate for this condition?

A) Venogram
B) CT scan of the lower legs
C) Ankle-brachial index (ABI)
D) PET scan
C) Ankle-brachial index (ABI)

A 55-year-old secretary with a recent history of breast cancer, for which she underwent surgery and radiation therapy, and a history of hypertension comes to your office for a routine checkup. Which of the following aspects of the physical are important to note when assessing the patient for peripheral vascular disease in the arms?

A) Femoral pulse, popliteal pulse
B) Dorsalis pedis pulse, posterior tibial pulse
C) Carotid pulse
D) Radial pulse, brachial pulse
D) Radial pulse, brachial pulse

You are a student in the vascular surgery clinic. You are asked to perform a physical examination on a patient with known peripheral vascular disease in the legs. Which of the following aspects is important to note when you perform your examination?

A) Size, symmetry, and skin color
B) Muscle bulk and tone
C) Nodules in joints
D) Lower extremity strength
A) Size, symmetry, and skin color

Swelling in the legs,
cyanosis, and lack of appropriate hair growth are all signs of peripheral vascular disease.

You are assessing a patient for peripheral vascular disease in the arms, secondary to a
complaint of increased weakness and a history of coronary artery disease and diabetes. You
assess the brachial and radial pulses and note that they are bounding. What does that translate toon a scale of 0 to 3?

A) 0
B) 3+
C) 2+
D) 1+
B) 3+

You are assessing a 59-year-old gas station owner for atherosclerosis in the lower
extremities. In which of the following locations would the patient’s pain make you concerned for this disease process?

A) Thigh
B) Knee
C) Calf
D) Ankle
C) Calf

Pain in the calf is the most common site for claudication

You are performing a routine check-up on an 81-year-old retired cotton farmer in the vascular surgery clinic. You note that he has a history of chronic arterial insufficiency. Which of the following physical examination findings in the lower extremities would be expected with this
disease?

A) Normal pulsation
B) Normal temperature
C) Marked edema
D) Thin, shiny, atrophic skin
D) Thin, shiny, atrophic skin

Thin, shiny, atrophic skin is more commonly seen in chronic arterial insufficiency;
in chronic venous insufficiency the skin often has a brown pigmentation and may be thickened.

A 77-year-old retired nurse has an ulcer on a lower extremity that you are asked to evaluate
when you do your weekly rounds at a local long-term care facility. All of the following are
responsible for causing ulcers in the lower extremities except for which condition?

A) Arterial insufficiency
B) Venous insufficiency
C) Diminished sensation in pressure points
D) Hypertension
D) Hypertension

Hypertension is not directly associated with the formation of ulcers. It is an indirect
risk factor if it is uncontrolled for a long time and associated with atherosclerosis, because it can
lead to arterial insufficiency or neuropathy.

As the internal diameter of a blood vessel changes, the resistance changes as well. Which
of the following descriptions depicts this relationship?

A) Resistance varies linearly with the diameter.
B) Resistance varies proportionally to the second power of the diameter.
C) Resistance varies proportionally to the third power of the diameter.
D) Resistance varies proportionally to the fourth power of the diameter.
D) Resistance varies proportionally to the fourth power of the diameter.

Mr. Edwards complains of cramps and difficulties with walking. The cramps occur in his calves consistently after walking about 100 yards. After a period of rest, he can start to walk again, but after 100 yards these same symptoms recur. Which of the following would suggest spinal stenosis as a cause of this pain?

A) Coldness and pallor of the legs
B) Relief of the pain with bending at the waist
C) Color changes of the skin
D) Swelling with tenderness of the skin
B) Relief of the pain with bending at the waist

Which of the following pairs of ischemic symptoms versus vascular supply is correct?

A) Lower calf/superficial femoral
B) Erectile dysfunction/iliac or pudendal
C) Buttock/common femoral
D) Upper calf/tibial or peroneal
B) Erectile dysfunction/iliac or pudendal

The ankle-brachial index (ABI) is calculated by dividing the systolic BP at the dorsalis
pedis by the systolic BP at the brachial artery. Which of the following values would be
consistent with mild peripheral arterial disease?

A) 1.1
B) 0.85
C) 0.65
D) 0.35
B) 0.85

The mild disease is represented by an ABI of 0.71 to 0.9. Any value above 0.9 is
normal. Moderate disease is defined as between 0.7 and 0.41, and severe disease is defined as
0.4 or less. Patients in the “severe” category have a 20% to 25% annual risk of death.

Asymmetric BPs are seen in which of the following conditions?

A) Coronary artery disease
B) Congenital narrowing of the aorta
C) Diffuse atherosclerosis
D) Vasculitis, as seen in systemic lupus erythematosus
B) Congenital narrowing of the aorta

Diminished radial pulses may be seen in patients with which of the following?

A) Aortic insufficiency
B) Hyperthyroidism
C) Arterial emboli
D) Early “warm” septic shock
C) Arterial emboli

Arterial emboli would decrease flow to a region, and therefore pulses would decrease as well. The other conditions actually cause bounding pulses.

When assessing temperature of the skin, which portion of your hand should be used?

A) Fingertips
B) Palms
C) Backs of fingers
D) Ulnar aspect of the hand
C) Backs of fingers

The backs of the fingers are thought to be the most temperature sensitive, perhaps
because the skin is thinnest there.

A patient presents with claudication symptoms and diminished pulses. Which of the
following is consistent with chronic arterial insufficiency?

A) Pallor of the foot when raised to 60 degrees for one minute
B) Return of color to the skin within 5 seconds of allowing legs to dangle
C) Filling of the veins of the ankles within 10 seconds of allowing the legs to dangle
D) Hyperpigmentation of the skin
A) Pallor of the foot when raised to 60 degrees for one minute

You note a painful ulcerative lesion near the medial malleolus, with accompanying
hyperpigmentation. Which of the following etiologies is most likely?

A) Arterial insufficiency
B) Neuropathic ulcer
C) Venous insufficiency
D) Trauma
C) Venous insufficiency

These features are most consistent with venous insufficiency. You may also see
scaling, redness, varicosities, and other findings. Arterial insufficiency usually affects distal or traumatized areas.

You are assessing a patient with joint pain and are trying to decide whether it is inflammatory or noninflammatory in nature. Which one of the following symptoms is consistent with an inflammatory process?

A) Tenderness
B) Cool temperature
C) Ecchymosis
D) Nodules
A) Tenderness

You are assessing a patient with diffuse joint pains and want to make sure that only the joints are the problem, and that the pain is not related to other diseases. Which of the following is
a systemic cause of joint pain?

A) Gout
B) Osteoarthritis
C) Lupus
D) Spondylosis
C) Lupus

Lupus is a systemic disease, one symptom of which may be joint pain. It is
important to consider the presence of a systemic illness when a patient presents with arthritis.

A 19-year-old college sophomore comes to the clinic for evaluation of joint pains. The student has been back from spring break for 2 weeks; during her holiday, she went camping. She notes that she had a red spot, shaped like a target, but then it started spreading, and then the joint pains started. She used insect repellant but was in an area known to have ticks. She has never been sick and takes no medications routinely; she has never been sexually active. What is the most likely cause of her joint pain?

A) Trauma
B) Gonococcal arthritis
C) Psoriatic arthritis
D) Lyme disease
D) Lyme disease

Lyme disease is characterized by a target-shaped red spot at the site of the bite, which disappears, then reappears and starts spreading (erythema migrans). Lyme disease can also
result in joint pain as well as cardiac and neurologic manifestations.

An 85-year-old retired housewife comes with her daughter to establish care. Her daughter is
concerned because her mother has started to fall more. As part of her physical examination, you
ask her to walk across the examination room. Which of the following is not part of the stance
phase of gait?

A) Foot arched
B) Heel strike
C) Mid-stance
D) Push-off
A) Foot arched

A 32-year-old warehouse worker presents for evaluation of low back pain. He notes a
sudden onset of pain after lifting a set of boxes that were heavier than usual. He also states that
he has numbness and tingling in the left leg. He wants to know if he needs to be off of work.
What test should you perform to assess for a herniated disc?

A) Leg-length test
B) Straight-leg raise
C) Tinel’s test
D) Phalen’s test
B) Straight-leg raise

The straight-leg raise involves having the patient lie supine with the examiner
raising the leg. If the patient experiences a sharp pain radiating from the back down the leg in an
L5 or S1 distribution, that suggests the presence of a herniated disc.

A 33-year-old construction worker comes for evaluation and treatment of acute onset of low
back pain. He notes that the pain is an aching located in the lumbosacral area. It has been present intermittently for several years; there is no known trauma or injury. He points to the left lower
back. The pain does not radiate and there is no numbness or tingling in the legs or incontinence. He was moving furniture for a friend over the weekend. On physical examination, you note
muscle spasm, with normal deep tendon reflexes and muscle strength. What is the most likely
cause of this patient’s low back pain?

A) Herniated disc
B) Compression fracture
C) Mechanical low back pain
D) Ankylosing spondylitis
) Mechanical low back pain

The case is an example of mechanical low back pain; in a large percentage of cases
there is no known underlying cause. The pain is often precipitated by moving, lifting, or twisting motions and relieved by rest.

A 50-year-old realtor comes to your office for evaluation of neck pain. She was in a motor vehicle collision 2 days ago and was assessed by the emergency medical technicians on site, but
she didn’t think that she needed to go to the emergency room at that time. Now, she has severe
pain and stiffness in her neck. On physical examination, you note pain and spasm over the
paraspinous muscles on the left side of the neck, and pain when you make the patient do active
range of motion of the cervical spine. What is the most likely cause of this neck pain?

A) Simple stiff neck
B) Aching neck
C) Cervical sprain
D) Cervical herniated disc
C) Cervical sprain

The patient most likely has an acute whiplash injury secondary to the collision. The
features of the physical examination, local tenderness and pain on movement, are consistent with
cervical sprain.

A 28-year-old graduate student comes to your clinic for evaluation of pain “all over.” With further questioning, she is able to relate that the pain is worse in the neck, shoulders, hands, low back, and knees. She denies swelling in her joints; she states that the pain is worse in the morning; there is no limitation in her range of motion. On physical examination, she has several points on the muscles of the neck, shoulders, and back that are tender to palpation; muscle
strength and range of motion are normal. Which of the following is likely the cause of her pain?

A) Rheumatoid arthritis
B) Osteoarthritis
C) Fibromyalgia
D) Polymyalgia rheumatica
C) Fibromyalgia

The patient has pain in specific trigger point areas on the muscles, with normal
strength and range of motion. This is an indication for fibromyalgia.

A 68-year-old retired banker comes to your clinic for evaluation of left shoulder pain. He swims for 30 minutes daily, early in the morning. He notes a sharp, catching pain and a sensation
of something grating when he tries overhead movements of his arm. On physical examination, you note tenderness just below the tip of the acromion in the area of the tendon insertions. The drop arm test is negative, and there is no limitation with shoulder shrug. The patient is not holding his arm close to his side, and there is no tenderness to palpation in the bicipital groove when the arm is at the patient’s side, flexed to 90 degrees, and then supinated against resistance. Based on this description, what is the most likely cause of his shoulder pain?

A) Rotator cuff tendinitis
B) Rotator cuff tear
C) Calcific tendinitis
D) Bicipital tendinitis
A) Rotator cuff tendinitis

Rotator cuff tendinitis is typically precipitated by repetitive motions, such as occurs
with throwing or swimming. Crepitus/grating is noted in the shoulder with range of motion.

A high school soccer player “blew out his knee” when the opposing goalie’s head and shoulder struck his flexed knee while the goalie was diving for the ball. All of the following structures were involved in some way in his injury, but which of the following is actually an
extra-articular structure?

A) Synovium
B) Joint capsule
C) Juxta-articular bone
D) Tendons
D) Tendons

Extra-articular structures include the periarticular ligaments, tendons, bursae, muscle, fascia, bone, nerve, and overlying skin. The articular structures include the joint capsule and articular cartilage, the synovium and synovial fluid, intra-articular ligaments, and juxta-articular bone.

Ray works a physical job and notes pain when he attempts to lift his arm over his head. When you move the shoulder passively, he has full range of motion without pain and there is no gross swelling or tenderness. What type of joint disease does this most likely represent?

A) Articular
B) Extra-articular
C) Neither
D) Both
B) Extra-articular

This description fits extra-articular disease. Articular disease typically involves swelling and tenderness of the entire joint and limits both active and passive range of motion. This is most likely extra-articular because it affects a certain portion of the range of motion, is
not painful with passive range of motion, and is not associated with gross swelling or tenderness

Mark is a contractor who recently injured his back. He was told he had a “bulging disc” to account for the burning pain down his right leg and slight foot drop. The vertebral bodies of the spine involve which type of joint?

A) Synovial
B) Cartilaginous
C) Fibrous
D) Synostosis
B) Cartilaginous

The vertebral bodies of the spine are connected by cartilaginous joints involving the
discs. The elbow would be an example of a synovial joint, and the sutures of the skull are an
example of a fibrous joint.

Which of the following synovial joints would be an example of a condylar joint?

A) Hip
B) Interphalangeal joints of the hand
C) Temporomandibular joint
D) Intervertebral joint
C) Temporomandibular joint

The TMJ is an example of a condylar joint because it involves the movement of two
surfaces which are not dissociable. The hip would be an example of a spheroidal joint and the interphalangeal joints of the hand are hinge joints. The intervertebral joints are not synovial
joints at all, but rather cartilaginous joints.

A 58-year-old man comes to your office complaining of bilateral back pain that now awakens him at night. This has been steadily increasing for the past 2 months. Which one of the following is the most reassuring in this patient with back pain?

A) Age over 50
B) Pain at night
C) Pain lasting more than 1 month or not responding to therapy
D) Pain that is bilateral
D) Pain that is bilateral

While bilateral pain can be associated with serious illness, it is not one of the “red
flags” of back pain. Red flags should make one suspicious for serious underlying systemic
disease such as cancer, infection, or others. This list includes: age over 50, history of cancer,
unexplained weight loss, pain lasting more than 1 month or not responding to treatment, pain at
night or increased by rest, history of intravenous drug use, or presence of infection. The
presence of one of these with low back pain indicates a 10% probability of a serious systemic
disease.

Marion presents to your office with back pain associated with constipation and urinary
retention. Which of the following is most likely?

A) Sciatica
B) Epidural abscess
C) Cauda equina
D) Idiopathic low back pain
C) Cauda equina

The presence of bowel and bladder symptoms associated with back pain is
worrisome and should suggest impingement of nerve roots S2-S4. For this reason idiopathic low back pain is unlikely. Epidural abscess may present with midline pain which can be
increased with percussion over the spinous processes. Sciatica is associated with pain which
radiates into the buttocks and/or down the posterior leg in the S1 distribution.

Louise, a 60-year-old, complains of left knee pain associated with tenderness throughout, redness, and warmth over the joint. Which of the following is least helpful in determining if a joint problem is inflammatory?

A) Tenderness
B) Pain
C) Warmth
D) Redness
B) Pain

Pain is present in both inflammatory and noninflammatory conditions. Warmth,
redness, and tenderness to palpation should lead one to consider an inflammatory etiology for the
pain.

Pain, swelling, loss of both active and passive motion, locking, and deformity would be consistent with which of the following?

A) Articular joint pain
B) Bursitis
C) Muscular injury
D) Nerve damage
A) Articular joint pain

These features are consistent with articular joint pain, whereas the other problems are associated with extra-articular structures.

You are working in a college health clinic and seeing a young woman with a red, painful,
swollen DIP joint on the left index finger. There are also a few papules, pustules, and vesicles on reddened bases, located on the distal extremities. This would be consistent with which of
the following?

A) Lyme disease
B) Systemic lupus erythematosus
C) Hives (urticaria)
D) Gonococcal arthritis
D) Gonococcal arthritis

The presentation of a monoarthritis in this age group should lead one to think of
gonococcal disease. Skin findings are often seen in conjunction with arthritis. Lyme disease is associated with an expanding erythematous patch. Lupus is associated with a “butterfly” rash
on the cheeks, while serum sickness and drug reactions can be associated with hives.

An obese 55-year-old woman went through menarche at age 16 and menopause 2 years ago. She is concerned because an aunt had severe osteoporosis. Which of the following is a risk factor for osteoporosis?

A) Obesity
B) Late menopause
C) Having an aunt with osteoporosis
D) Delayed menarche
D) Delayed menarche

Obesity and late menopause are not associated with osteoporosis. Having a first-degree relative with osteoporosis is a risk factor, but an aunt is a second-degree relative.
Delayed menarche is the only choice which is a known risk factor for osteoporosis.

A 38-year-old woman comes to you and has multiple small joints involved with pain, swelling, and stiffness. Which of the following is the most likely explanation?

A) Rheumatoid arthritis
B) Septic arthritis
C) Gout
D) Trauma
A) Rheumatoid arthritis

Rheumatoid arthritis is a systemic disease and accounts for multiple symmetrically
involved joints. Septic arthritis is usually monoarticular, as are gout and trauma-related joint pain.

Mrs. Fletcher comes to your office with unilateral pain during chewing, which is chronic.
She does not have facial tenderness or tenderness of the scalp. Which of the following is the
most likely cause of her pain?

A) Trigeminal neuralgia
B) Temporomandibular joint syndrome
C) Temporal arteritis
D) Tumor of the mandible
B) Temporomandibular joint syndrome

Temporomandibular joint syndrome is a very common cause of pain with chewing.
Ischemic pain with chewing, or jaw claudication, can occur with temporal arteritis, but the lack
of tenderness of the scalp overlying the artery makes this less likely. Trigeminal neuralgia can
be associated with extreme tenderness over the branches of the trigeminal nerve. While a tumor
of the mandible is possible, is it much less likely than the other choices.

A man’s wife is upset because when she hugs him with her hands on his left shoulder blade, “It feels creepy.” This came on gradually after a recent severe left-sided rotator cuff tear. How long does it usually take to develop muscular atrophy with increased prominence of the scapular spine following a rotator cuff tear?

A) 1 week
B) 2-3 weeks
C) 1 month
D) 2-3 months
B) 2-3 weeks

Prominence of the scapular spine occurs with generalized muscle wasting as well as with specific injuries such as a rotator cuff tear. It is easily palpable, even through indoor
clothing, although the back should be exposed to make other important observations. Atrophy usually occurs several weeks following a rotator cuff tear.

Phil comes to your office with left “shoulder pain.” You find that the pain is markedly worse when his left arm is drawn across his chest (adduction). Which of the following would you suspect?

A) Rotator cuff tear
B) Subacromial bursitis
C) Acromioclavicular joint involvement
D) Adhesive capsulitis
C) Acromioclavicular joint involvement

Adduction of the patient’s arm across his chest can cause pain if the
acromioclavicular joint is involved. In adhesive capsulitis, this maneuver may not be possible
due to limited range of motion. Subacromial bursitis would present with tenderness inferior to
the acromion. Rotator cuff injury would ordinarily not be associated with pain during this
maneuver.

Two weeks ago, Mary started a job which requires carrying 40-pound buckets. She presents with elbow pain worse on the right. On examination, it hurts her elbows to dorsiflex her hands against resistance when her palms face the floor. What condition does she have?

A) Medial epicondylitis (golfer’s elbow)
B) Olecranon bursitis
C) Lateral epicondylitis (tennis elbow)
D) Supracondylar fracture
C) Lateral epicondylitis (tennis elbow)

Mary’s injury probably occurred by lifting heavy buckets with her palms down
(toward the bucket). This caused her chronic overuse injury at the lateral epicondyle. Medial
epicondylitis has reproducible pain when palmar flexion against resistance is performed and also
features tenderness over the involved epicondyle. Olecranon bursitis produces erythema and swelling over the olecranon process. A supracondylar fracture of the humerus is a major injury
and would present more acutely.

A high school football player injured his wrist in a game. He is tender between the two
tendons at the base of the thumb. Which of the following should be considered?

A) DeQuervain’s tenosynovitis
B) Scaphoid fracture
C) Wrist sprain
D) Rheumatoid arthritis
B) Scaphoid fracture

The “anatomic snuffbox” is found between the extensor and abductor tendons at the base of the thumb. Tenderness should make one think of a scaphoid fracture. Not only is this the most common carpal bone injury, but the poor blood supply puts the bone at risk for
avascular necrosis when injured. This fracture is commonly missed on x-ray, so this is an important physical finding to support further or repeated studies.

Mrs. Fletcher complains of numbness of her right hand. On examination, sensation of the
volar aspect of the web of the thumb and index finger and the pulp of the middle finger are
normal. The pulp of the index finger has decreased sensation. Which of the following is
affected?

A) Median nerve
B) Ulnar nerve
C) Radial nerve
A) Median nerve

The pulp of the index finger is innervated by the median nerve. A decrease in
sensation at this area would support a diagnosis of carpal tunnel syndrome. The pulp of the fifth finger is supplied by the ulnar nerve, and the dorsal web space of the thumb and index finger is supplied by the radial nerve.

A 50-year-old woman presents with “left hip pain” of several weeks duration. There is
marked tenderness when you press over her proximal lateral thigh. What do you think she has?

A) Osteoarthritis
B) Rheumatoid arthritis
C) Sciatica
D) Trochanteric bursitis
D) Trochanteric bursitis

Bursitis is usually accompanied by tenderness on examination. This location is
consistent with trochanteric bursitis. Osteoarthritis would generally not be tender and would
more likely have decreased range of motion. Rheumatoid arthritis and sciatica would not likely be tender over this area.

Sarah presents with left lateral knee pain and has some locking in full extension. There is
tenderness over the medial joint line. When the knee is extended with the foot externally rotated
and some valgus stress is applied, a click is noted. What is the most likely diagnosis?

A) Torn anterior cruciate ligament
B) Torn posterior cruciate ligament
C) Torn medial meniscus
D) Torn lateral meniscus
C) Torn medial meniscus

This maneuver is called the McMurray test. Along with the medial joint line
tenderness, you should suspect a medial meniscus injury. Although we can’t rule out a lateral meniscus tear, the
tenderness along the medial joint line makes this the more likely site of injury.

A 28-year-old book editor comes to your clinic, complaining of strange episodes. He states
that about once a week for the last 3 months his left hand and arm will stiffen and then start jerking. He says that after a few seconds his whole left arm and then his left leg will also start to jerk. He denies any loss of consciousness or loss of bowel or bladder control. When the symptoms resolve, his arm and leg feel tired but otherwise he feels fine. His past medical history
is significant for a cyst in his brain that was removed 6 months ago. He is married and has two
children. His parents are both healthy. On examination you see a scar over the right side of his
head but otherwise his neurologic examination is unremarkable.
What type of seizure disorder is he most likely to have?

A) Generalized tonic-clonic seizure
B) Generalized absence seizure
C) Simple partial seizure (Jacksonian)
D) Complex partial seizure
C) Simple partial seizure (Jacksonian)

Simple partial seizures start with a unilateral symptom, involve no loss of
consciousness, and have a normal postictal state. In a Jacksonian seizure the symptoms start with one body part and “march” along the same side of the body.

A 7-year-old child is brought to your clinic by her mother. The mother states that her daughter is doing poorly in school because she has some kind of “ADD”. You ask the mother what makes her think the child has ADD. The mother tells you that both at home and at school her daughter will just zone out for several seconds and lick her lips. She states it happens at least four to six times an hour. She says this has been happening for about a year. After several seconds of lip-licking her daughter seems normal again. She states her daughter has been generally healthy with just normal childhood colds and ear infections. The
patient’s parents are both healthy and no other family members have had these symptoms.
What type of seizure disorder is she most likely to have?

A) Generalized tonic-clonic seizure
B) Generalized absence seizure
C) Simple partial seizure (Jacksonian)
D) Complex partial seizure
B) Generalized absence seizure

In an absence seizure there is no tonic-clonic activity. There is a sudden, brief lapse of consciousness with blinking, staring, lip-smacking, or hand movements that resolve quickly to
full consciousness. It is easily mistaken for daydreaming or ADD.

A 37-year-old insurance agent comes to your office, complaining of trembling hands. She says that for the past 3 months when she tries to use her hands to fix her hair or cook they shake badly. She says she doesn’t feel particularly nervous when this occurs but she worries that other people will think she has an anxiety disorder or that she’s a drinker. She admits to having some recent fatigue, trouble with vision, and difficulty maintaining bladder control. Her past medical history is remarkable for hypothyroidism. Her mother has lupus and her father is healthy. She has an older brother with type 1 diabetes. She is married and has three children. She denies
tobacco, alcohol, or drug use. On examination, when she tries to reach for a pencil to fill out the
health form she has obvious tremors in her dominant hand.
What type of tremor is she most likely to have?

A) Resting tremor
B) Postural tremor
C) Intention tremor
C) Intention tremor

Intention tremors are absent at rest or in a postural position and occur only with
intentional movement of the hands. This is seen in cerebellar disease (stroke or alcohol use) or in
multiple sclerosis. This patient’s tremor, fatigue, bladder problems, and visual problems are
suggestive of multiple sclerosis.

A 77-year-old retired school superintendent comes to your office, complaining of unsteady
hands. He says that for the past 6 months, when his hands are resting in his lap they shake uncontrollably. He says when he holds them out in front of his body the shaking diminishes, and when he uses his hands the shaking is also better. He also complains of some difficulty getting up out of his chair and walking around. He denies any recent illnesses or injuries. His past medical history is significant for high blood pressure and coronary artery disease, requiring a stent in the past. He has been married for over 50 years and has five children and 12 grandchildren. He denies any tobacco, alcohol, or drug use. His mother died of a stroke in her
70s and his father died of a heart attack in his 60s. He has a younger sister who has arthritis problems. His children are all essentially healthy. On examination you see a fine, pill-rolling tremor of his left hand. His right shows less movement. His cranial nerve examination is normal. He has some difficulty rising from his chair, his gait is slow, and it takes him time to turn around to walk back toward you. He has almost no “arm swing” with his gait.
What type of tremor is he most likely to have?

A) Resting tremor
B) Postural tremor
C) Intention tremor
A) Resting tremor

Resting tremors occur when the hands are literally at rest, such as sitting in the lap. These are slow, fine tremors, such as the pill-rolling seen in Parkinson’s disease, which this patient most likely has. Decreased arm swing with ambulation is one of the earliest objective findings of Parkinson’s disease.

A 48-year-old grocery store manager comes to your clinic, complaining of her head being
“stuck” to one side. She says that today she was doing her normal routine when it suddenly felt
like her head was being moved to her left and then it just stuck that way. She says it is somewhat
painful because she cannot get it moved back to normal. She denies any recent neck trauma. Her
past medical history consists of type 2 diabetes and gastroparesis. She is on oral medication for each. She is married and has three
children. She denies tobacco, alcohol, or drug use. Her father has diabetes and her mother passed away from breast cancer. Her children are healthy. On examination you see a slightly overweight Hispanic woman appearing her stated age. Her head is twisted grotesquely to her left but otherwise her examination is normal.
What form of involuntary movement does she have?

A) Chorea
B) Asbestosis
C) Tic
D) Dystonia
D) Dystonia

Dystonia involves large movements of the body, such as with the head or trunk, leading to grotesque twisted postures. Some medications (such as one commonly used for gastroparesis) often cause dystonia.

A 41-year-old real estate agent comes to your office, complaining that he feels like his face is paralyzed on the left. He states that last week he felt his left eyelid was drowsy and as the day
progressed he was unable to close his eyelid all the way. Later he felt like his smile became
affected also. He denies any recent injuries but had an upper respiratory viral infection last month. His past medical history is unremarkable. He is divorced and has one child. He smokes one pack of cigarettes a day, occasionally drinks alcohol, and denies any illegal drug use. His
mother has high blood pressure and his father has sarcoidosis. On examination you ask him to close his eyes. He is unable to close his left eye. You ask him to open his eyes and raise his eyebrows. His right forehead furrows but his left remains flat. You then ask him to give you a big smile. The right corner of his mouth raises but the left side of his mouth remains the same.
What type of facial paralysis does he have?
A) Peripheral CN VII paralysis
B) Central CN VII paralysis
A) Peripheral CN VII paralysis

In a peripheral lesion the entire side of the face will be involved. This causes the
inability to close the eye, raise the eyebrow, wrinkle the forehead, and smile on the affected side.
Bell’s palsy is an example of this type of paralysis and is probably what is affecting this patient.

A 60-year-old retired seamstress comes to your office, complaining of decreased sensation in her hands and feet. She states that she began to have the problems in her feet a year ago but now it has started in her hands also. She also complains of some weakness in her grip. She has had no recent illnesses or injuries. Her past medical history consists of having type 2 diabetes for 20 years. She now takes insulin and oral medications for her diabetes. She has been married for 40 years. She has two healthy children. Her mother has Alzheimer’s disease and coronary artery disease. Her father died of a stroke and also had diabetes. She denies any tobacco, alcohol, or drug use. On examination she has decreased deep tendon reflexes in the patellar and Achilles tendons. She has decreased sensation of fine touch, pressure, and vibration on both feet. She has decreased two-point discrimination on her hands. Her grip strength is decreased and her plantar and dorsiflexion strength is decreased.
Where is the disorder of the peripheral nervous system in this patient?

A) Anterior horn cell
B) Spinal root and nerve
C) Peripheral polyneuropathy
D) Neuromuscular junction
C) Peripheral polyneuropathy

With peripheral polyneuropathy there will be distal extremity symptoms before proximal symptoms. There will be weakness and atrophy and decreased sensory sensations. There is often the classic glove-stocking distribution pattern of the lower legs and hands. Causes include diabetic neuropathy, as in this case, alcoholism, and vitamin deficiencies.

A 21-year-old engineering student comes to your office, complaining of leg and back paincand of tripping when he walks. He states this started 3 months ago with back and buttock paincbut has since progressed to feeling weak in his left leg. He denies any bowel or bladdercsymptoms. He can think of no specific traumatic incidences but he was a defensive lineman in high school and junior college. His past medical history is unremarkable. He denies tobacco use or alcohol or drug abuse. His parents are both healthy. On examination he is tender over the
lumbar spine and he has a positive straight-leg raise on the left. His Achilles tendon deep reflex
is decreased on the left. While watching his gait you notice he has to pick his left foot up high in
order not to trip.
What abnormality of gait does he most likely have?

A) Sensory ataxia
B) Parkinsonian gait
C) Steppage gait
D) Spastic hemiparesis
C) Steppage gait

This gait is associated with foot drop, usually secondary to a lower motor neuron
disease. This is often seen with a herniated disc, such as in this patient.

A 17-year-old high school student is brought in to your emergency room in a comatose state. His friends have accompanied him and tell you that they have been shooting up heroin tonight
and they think their friend may have had too much. The patient is unconscious and cannot protect his airway, so he is intubated. His heart rate is 60 and he is breathing through the ventilator. He is not posturing and he does not respond to a sternal rub. Preparing to finish the neurologic examination, you get a penlight.
What size pupils do you expect to see in this comatose patient?

A) Pinpoint pupils
B) Large pupils
C) Asymmetric pupils
D) Irregularly shaped pupils
A) Pinpoint pupils

Narcotics and cholinergics cause very small (1 mm) pupils. Reactions to light can be
appreciated with a magnifying glass.

A 37-year-old woman is brought into your emergency room comatose. The paramedics say
her husband found her unconscious in her home. Her past medical history consists of type 1
diabetes and she is on insulin. In the ambulance the paramedics obtained a glucose check and her
sugar was 15. They began a dextrose saline infusion and intubated her to protect her airway. Despite their efforts, she is posturing in the emergency room with her arms straight at her side and her jaw clenched. Her legs are also straight and her feet are plantar flexed.
What type of posturing is she showing?

A) Decorticate rigidity
B) Decerebrate rigidity
C) Hemiplegia
D) Chorea
B) Decerebrate rigidity

In this type of rigidity the jaws are clenched and the neck is extended. The arms are adducted and stiffly extended at the elbows with forearms pronated and wrists and fingers flexed.
The legs are stiffly extended at the knees with the feet plantar flexed. This posture occurs with
lesions in the diencephalon, midbrain, or pons. It can also be seen with severe metabolic disorder such as hypoxia or hypoglycemia, as in this case.

A patient presents with a left-sided facial droop. On further testing, you note that he is unable to wrinkle his forehead on the left and has decreased taste. Which of the following is true?

A) This represents a central lesion.
B) This represents a CN IV lesion.
C) This may be related to travel.
D) This most likely represents a stroke.
C) This may be related to travel.

Because the forehead is also involved, this represents a peripheral nerve lesion of CN VII and does not represent a classic middle cerebral artery stroke. The latter would spare the upper face but include speech difficulties as well as upper extremity weakness on the
ipsilateral side. One cause of this type of lesion is Lyme disease and relates to travel to endemic areas, so a careful travel history should be sought.

Which is true of examination of the olfactory nerve?

A) It is not tested for laterality.
B) The smell must be identified to declare a normal response.
C) Abnormal responses may be seen in otherwise normal elderly.
D) Allergies are unrelated to testing of this nerve.
C) Abnormal responses may be seen in otherwise normal elderly.

Abnormal olfactory nerve examination findings may be seen in otherwise normal
elderly but may also be associated with other conditions such as Parkinson’s disease. You should try to determine if only one side is abnormal by occluding the contralateral nostril. The smell must only be detected, not identified by name, to indicate a normal examination. If nasal
occlusion occurs for other reasons, such a allergic rhinitis or anatomic abnormalities, the nerve cannot be tested and may seem to be abnormal for unrelated reasons.

Steve has had a stroke and comes to you for follow-up today. On examination you find that he has increased muscle tone, some involuntary movements, an abnormal gait, and a
slowness of response in movements. He most likely has involvement of which of the
following?

A) The corticospinal tract
B) The cerebellum
C) The cerebrum
D) The basal ganglia
D) The basal ganglia

You are conducting a mental status examination and note impairment of speech and
judgement, but the rest of your examination is intact. Where is the most likely location of the
problem?

A) Cerebrum
B) Cerebellum
C) Brainstem
D) Basal ganglia
A) Cerebrum

The cerebrum is responsible for higher cognitive functions such as speech and
judgement.

A patient presents with a daily headache which has worsened over the past several months.
On funduscopic examination, you notice that the disk edge is indistinct and the veins do not
pulsate. Which is most likely?

A) Migraine
B) Glaucoma
C) Visual acuity problem
D) Increased intracranial pressure
D) Increased intracranial pressure

This is a description of papilledema, which should make you think of increased
intracranial pressure. This can be a critical finding. This patient may have a brain tumor or benign intracranial hypertension.

A young woman comes in today, complaining of fatigue, irregular menses, and polyuria which have gradually increased over the past few months. Which eye findings would be consistent with her condition?

A) An upper quadrantanopsia
B) A lower quadrantanopsia
C) A bitemporal hemianopsia
D) An increased cup-to-disc ratio
C) A bitemporal hemianopsia

These symptoms are consistent with a pituitary lesion. Enlargement of a tumor in this area would compress the fibers responsible for the lateral visual fields. A quadrantanopsia
would usually be caused by a lesion in the optic radiations in the parietal lobe of the cerebrum.
Glaucoma would cause a narrowing of the entire visual field, not just the lateral aspects.

A patient with a history of seizure disorder and on several seizure medications says a friend noted “jumping eye movements.” The patient describes a sensation of movement at rest since his medications were adjusted upward following a breakthrough seizure several weeks ago. On
examination you note that the eyes both slowly move to the right and then quickly jump to the
left. Which of the following is true?

A) This is called nystagmus to the left
B) This is called saccadic eye movement
C) This represents a subclinical seizure
D) This most likely has an ominous cause
A) This is called nystagmus to the left

Nystagmus is named for the fast component, in this case, toward the left.
Nystagmus is common with several seizure medications and in this case is likely due to the
recent increase in medications rather than a more ominous cause.

You are testing the biceps strength in a young man following a spinal trauma from a motor vehicle accident. He cannot lift his hand upward, but if the arm is abducted to 90 degrees, he can then move his forearm side to side. This would represent which muscle strength grading?

A) I
B) II
C) III
D) IV
B) II

The ability to move an extremity, but not against gravity, represents a strength of 2 out of 5. Zero represents no muscular contraction detected (not even a “flicker”); one
represents a contraction but no movement of the extremity; three means that the extremity can
move against gravity but not against resistance; four means perceived weakness but the patient can oppose some resistance; and five is normal.

You ask a patient to hold her arms up, with her palms up, and then to close her eyes. The right arm begins to move downward after a few seconds and her thumb rotates upward. This is most likely a problem with which part of the nervous system?

A) Corticospinal tract
B) Spinothalamic tract
C) Thalamus
D) Dorsal root ganglion
A) Corticospinal tract

This describes a pronator drift, which signifies decreased position sense
involvement of the corticospinal tract. This tract does not travel through the thalamus. This is
commonly tested as an early sign of stroke. This would not occur with a dorsal root ganglion
problem.

You are examining a child with severe cerebral palsy. When you suddenly move his foot
dorsally, a sustained “beating” of the foot against your hand ensues. What does this represent?

A) A focal seizure
B) Clonus
C) Extinction
D) Reinforcement
B) Clonus

Clonus is a sustained rhythmic “beating” which correlates with CNS disease and
hyperreflexia. A focal seizure could be virtually ruled out by stopping the stimulus and watching the phenomenon stop. Extinction is a term applied to sensory testing where one side of a simultaneous, bilateral stimulus is not felt because of damage to the cortex. Reinforcement
applies to enhancing reflex examination by distracting the patient, for example, by pulling his
hands against each other.

Jim is an HIV-positive patient who complains about back pain in addition to several other
problems. On percussion, there is slight tenderness over the T7 vertebrae, and when you flex
his thigh to 90 degrees and extend his lower legs, you meet strong resistance at about 45 degrees
of extension. What are likely causes of this constellation of symptoms?

A) Fractured vertebrae
B) Malingering
C) Infection
D) Medication side effect
C) Infection

This represents Kernig’s sign. When present bilaterally it often indicates
meningeal irritation.

A patient with alcoholism is brought in with confusion. You ask him to “stop traffic” with his palms and notice that every few seconds his palms suddenly move toward the floor.
What does this indicate?

A) Stroke
B) Metabolic problems
C) Carpal tunnel syndrome
D) Severe fatigue and weakness
B) Metabolic problems

This is asterixis and represents the inability to maintain a sustained contraction of the muscles. It is usually due to various metabolic diseases. Most would consider checking an ammonia level in this patient.

You examine a “sleepy” patient. You note that she will open her eyes and look at you but
responds slowly and is confused. She does not appear interested in her surroundings. How would you describe her level of consciousness?

A) Lethargic
B) Obtunded
C) Stuporous
D) Comatose
B) Obtunded

A woman experiences syncope after hearing that her son was severely injured. She becomes pale and collapses to the ground without injuring herself. On waking, she states that she feels very warm. She denies any other symptoms. There are no findings on examination.
What caused her loss of consciousness?

A) Micturition syncope
B) Postural hypotension
C) Cardiac arrhythmia
D) Vasovagal syncope
D) Vasovagal syncope

This is a classic description of vasodepressor or vasovagal syncope with the feeling
of warmth, while bystanders note paleness. The lack of injury is also helpful because she has
maintained her protective reflexes. Injuring oneself can indicate that a cardiac origin for
syncope may be present. Micturition syncope occurs with urination, and there are no postural
changes mentioned, making postural hypotension unlikely.

A 7-year-old boy is performing poorly in school. His teacher is frustrated because he is
frequently seen “staring off into space” and not paying attention. If this is a seizure, it most likely represents which type?

A) Pseudoseizure
B) Tonic-clonic seizure
C) Absence
D) Myoclonus
C) Absence

A patient comes to you because she is experiencing a tremor only when she reaches for things. This becomes worse as she nears the “target.” When you ask her to hold out her hands, no tremor is apparent. What type of tremor does this most likely represent?

A) Intention tremor
B) Postural tremor
C) Resting tremor
D) Nervous tremor
A) Intention tremor

A young woman comes in with brief, rapid, jerky, irregular movements. They can occur
at rest or during other intentional movements and involve mostly her face, head, lower arms, and
hands. How would you describe these movements?

A) Tics
B) Dystonia
C) Athetosis
D) Chorea
D) Chorea

These represent chorea because they are brief, rapid, unpredictable, and irregular. Tics are irregular but tend to be stereotyped and can be vocal (throat-clearing), facial
expressions, or shoulder shrugging. Athetosis is a slow, squirming motion usually affecting the
face and distal extremities. Dystonia is similar to athetosis but the movements are more coarse
and can involve twisted postural changes.

Which of the following changes are expected in vision as part of the normal aging process?

A) Cataracts
B) Glaucoma
C) Macular degeneration
D) Blurring of near vision
D) Blurring of near vision

The lens loses its elasticity over time as part of the normal aging process, and the eye is less able to accommodate and focus on near objects; therefore, the patient will be expected to have blurring of near vision.

A 72-year-old retired truck driver comes to the clinic with his wife for evaluation of hearing loss. He has noticed some decreased ability to hear what his wife and grandchildren are saying to him. He admits to lip-reading more. He has a history of noise exposure in his young adult years: He worked as a sound engineer at a local arena and had to attend a lot of concerts. Based on this
information, what is the most likely finding regarding his hearing acuity?

A) Loss of acuity for middle-range sounds
B) Increase of acuity for low-range sounds
C) Loss of acuity for high-range sounds
D) Increase of acuity for high-range sounds
A) Loss of acuity for middle-range sounds

A 79-year-old retired banker comes to your office for evaluation of difficulty with urination; he gets up five to six times per night to urinate and has to go at least that often in the daytime. He does not feel as if his bladder empties completely; the strength of the urinary stream is
diminished. He denies dysuria or hematuria. This problem has been present for several years but
has worsened over the last 8 months. You palpate his prostate. What is your expected physical
examination finding, based on this description?

A) Normal size, smooth
B) Normal size, boggy
C) Enlarged size, smooth
D) Enlarged size, boggy
C) Enlarged size, smooth

Leave a Comment

Scroll to Top